Вы находитесь на странице: 1из 150

Apuntes de

Física con
Ejercicios
Resueltos
Ingreso a
Medicina UNA

Instituto Grigory
Perelman Prof. Master Emilio Ortiz
Eligio Ayala 844 casi Trepowski
Tacuary
Tomados del libro de Frank J. Blatt con solución de
441-320 Celular (0971) ejercicios del libro y de exámenes anteriores.
329061
También del libro de Bonjorno.
emiliortiz1

hotmail.com

08/05/2010
Instituto Grigory Perelman. Prof. Master Emilio Ortiz
Trepowski. Teléfono 441.320. emiliortiz1@hotmail.com.

Introducción
La física es una de las ciencias más básicas. Trata del comportamiento y la estructura de la
materia. El campo de la física está usualmente dividido en las áreas de movimiento, fluidos,
calor, sonido, luz, electricidad y magnetismo, y los tópicos modernos de relatividad, estructura
atómica, física de la materia condensada, física nuclear, partículas elementales, y astrofísica.
Comenzaremos con el movimiento (o mecánica, como es usualmente llamada).

Unidades, Dimensiones, Vectores y Otros Preliminares

Unidades, estándares, y el SI sistema


Algunas longitudes típicas o distancias (orden de magnitud)
Longitud (o distancia) Metros (aproximado)

Neutrón o protón (radio) 10 15


m

Átomo 10 10
m

Viruses 10 7 m
Hoja de papel (finura) 10 4 m

Ancho de un dedo 10 2 m
Largo de un campo de fútbol 102 m

Altura del Monte Everest 104 m


Diámetro de la tierra 107 m

De la tierra a la luna 1011 m


La estrella más cercana 1016 m
La galaxia más cercana 1022 m

La galaxia más lejana visible 1026 m

2
Instituto Grigory Perelman. Prof. Master Emilio Ortiz
Trepowski. Teléfono 441.320. emiliortiz1@hotmail.com.

Algunos intervalos de tiempo típicos

Intervalo de tiempo Segundos (aproximado)

Vida de una partícula muy inestable 10 23


s

Vida de elementos radioactivos 10 22


s a 1028 s

Vida de un muon 10 6 s

Tiempo entre latidos cardíacos humano 100 ( 1 s)

Un día 105 s

Un año 3 107 s

La vida de un ser humano en tiempo 2 109 s

Longitud de la historia grabada 1011 s

Humanos sobre la tierra 1014 s

Vida sobre la tierra 1017 s

Edad del universo 1018 s

Algunas masas

Objeto Kilogramos (aproximadamente)

Electrón 10 30
kg

Protón, neutrón 10 27
kg

DNA molécula 10 17
kg

Bacteria 10 15
kg

Mosquito 10 5 kg

3
Instituto Grigory Perelman. Prof. Master Emilio Ortiz
Trepowski. Teléfono 441.320. emiliortiz1@hotmail.com.

Plum 10 1 kg

Persona 102 kg

Barco 108 kg

Tierra 6 1024 kg

Sol 2 1030 kg

Galaxia 10 41 kg

Prefijos del (SI) métrico


Prefijo Abreviatura Valor

exa E 1018

peta P 1015

tera T 1012

giga G 109

mega M 106

kilo k 103

hecto h 102

deka da 101

deci d 10 1

centi c 10 2

mili m 10 3

micro 10 6

nano n 10 9

4
Instituto Grigory Perelman. Prof. Master Emilio Ortiz
Trepowski. Teléfono 441.320. emiliortiz1@hotmail.com.

pico p 10 12

femto f 10 15

atto a 10 18

Unidades
En el Sistema Internacional de Medidas , o unidades SI, el metro, el kilogramo, y el segundo
son las unidades fundamentales de la longitud, masa, y tiempo, respectivamente.

Ejemplo 1.1.
Un auto se desplaza a una velocidad de 50 millas por hora (mph). ¿Cuál es la velocidad del auto
en kilómetros por hora y en metros por segundo?

Solución.
La conversión entre millas y kilómetros es 1 milla 1, 61 km . Denotando la velocidad del auto
por v , tenemos que

50 millas 1, 61 km km
v 80,5
1 hora 1 milla hora

Observe que las unidades, millas, se cancelan en la conversión.

Para convertir a metros por segundo, observamos que hay una hora por cada 60 minutos y un
minuto cada 60 segundos, y que hay 1.000 metros por kilómetro. Así

km 80,5 km 1h 1 min 1.000 m m


80, 5 22, 4
h 1h 60 min 60 s 1 km s

Ejemplo 1.2.
¿Cuál es el factor de conversión entre pies cúbico y litros?

Solución.

5
Instituto Grigory Perelman. Prof. Master Emilio Ortiz
Trepowski. Teléfono 441.320. emiliortiz1@hotmail.com.

Un litro ( L ) está definido como 1.000 cm3 . Para obtener la respuesta, debemos por lo tanto
primero determinar el número de centímetros cúbicos contenidos en un pies cúbico. Dado que
1 ft 30, 48 cm , se deduce que

3 3
1 ft 30, 48 cm 28.320 cm3 28,32 L

Una de las características atractivas y convenientes del SI es que es un sistema decimal. Los
kilómetros, microgramos, nanosegundos, megawatts, son todos derivados desde las unidades
básicas mediante la multiplicación por potencias enteras de diez.

Unidades Fundamentales
Las fuerzas, velocidades, presiones, energías – ciertamente todas las propiedades mecánicas-
pueden ser expresadas en términos de tres cantidades básicas: masa, longitud, y tiempo. En el
SI las unidades correspondientes son

Kilogramo para la masa, metro para la longitud, y segundo para el tiempo.

Tabla 1.2. Prefijos y sus símbolos usados para designar los múltiplos decimales y submúltiplos

Factor Prefijo Símbolo

1018 exa E

1015 peta P

1012 tera T

109 giga G

106 mega M

103 kilo k

102 hecto h

101 deca da

10 1 deci d

10 2 centi c

10 3 mili m

6
Instituto Grigory Perelman. Prof. Master Emilio Ortiz
Trepowski. Teléfono 441.320. emiliortiz1@hotmail.com.

10 6 micro

10 9 nano n

10 12 pico p

10 15 femto f

10 18 atto a

Unidades Derivadas y Análisis Dimensional


Las cantidades que son importantes para los científicos no están limitadas a la masa, la
longitud, y el tiempo. A menudo describimos el comportamiento de los objetos en términos de
sus velocidades; necesitamos identificar las fuerzas que actúan sobre los cuerpos; prestamos
atención a las energías consumidas por los implementos y nos da curiosidad la potencia de un
motor; la presión atmosférica es un indicador útil de las condiciones del tiempo. Todas estas
aparentemente dispares propiedades, medidas en las unidades de metros por segundo
(velocidad), newton (fuerza), joule (energía), watt (potencia) y pascal (presión), son
últimamente expresadas como productos de potencias de masa, longitud y tiempo. Estas
unidades por lo tanto son conocidas como unidades derivadas, para distinguirlas de las
unidades más fundamentales.

La especificación numérica de una cantidad particular, una distancia o velocidad, por ejemplo,
depende del sistema de unidades que empleamos. Por ejemplo, como se demostró en el
Ejemplo 1.1. , un auto que viaja a 50 mph, 80,5 km por hora, y 22,4 m por s son todas las
mismas velocidades. Pero notemos que la combinación de unidades usadas para caracterizar la
velocidad es la misma siempre, llámese, el ratio longitud/tiempo. El tipo de unidad involucrada
es llamada dimensión de la variable y no depende del sistema de unidades que es usado.
Siempre usaremos corchetes, , para indicar la dimensión de una variable. La dimensión de
L 3
velocidad es , la de distancia es L . La dimensión de volumen es L . Así cuando
L
decimos que un auto obtiene 30 millas por galón, la dimensión de esta variable es
L 2
3
L .
L

Las ecuaciones que relacionan varias cantidades físicas deben ser dimensionalmente
homogéneas. Por esto nosotros significamos lo siguiente. Si una ecuación establece

7
Instituto Grigory Perelman. Prof. Master Emilio Ortiz
Trepowski. Teléfono 441.320. emiliortiz1@hotmail.com.

A B C

los términos A, B, C deben todos tener la misma dimensión.

Ejemplo 1.3.
En un momento dado, que nosotros identificamos como el tiempo t 0, un auto está en
50 m al este de un punto inicial y se está moviendo hacia el este a una velocidad constante de
m
10 . ¿Qué tan lejos está desde su punto inicial en t 4s ?
s

Solución.
Hagamos que la distancia desde el punto inicial sea designada por d . Sabemos que cuando
t 0 s, d 50m; llamamos a esa distancia d0 . También sabemos que en la medida que el
tiempo progresa, d se incrementa en la medida que el auto se mueve más al este a una
velocidad constante. La distancia adicional que el auto viaja depende de su velocidad y del
tiempo que transcurre. Así esperamos que d esté dada por una ecuación de la forma:

d d0 X

Donde X es alguna combinación algebraica de velocidad y tiempo que debe tener la


dimensión de longitud que satisface la condición de homogeneidad condicional.

L
Dado que la dimensión de velocidad es y que la del tiempo es T , la única combinación
L
de velocidad, v, y tiempo, t , que tendrá la dimensión de longitud es el producto vt. Por lo
tanto,

d d0 Avt

debe ser la expresión correcta. Aquí A es una constante numérica sin dimensión. El análisis
dimensional no nos puede decir cuanto será el valor de esta constante. En este caso, si las
unidades para d , d 0 , v y t son consistentes, por ejemplo, metros, metros por segundos, y
segundos, la constante A=1. Sin embargo, si d y d0 están en metros, v , en millas por hora, y t
en segundos, la constante A es 0,444, el factor de conversión de millas por hora a metros por
segundo.

d 50m 10m / s 4 s 90m.

Escalares y Vectores
8
Instituto Grigory Perelman. Prof. Master Emilio Ortiz
Trepowski. Teléfono 441.320. emiliortiz1@hotmail.com.

Definiremos algunas relaciones trigonométricas que nos serán de utilidad:

a
sin
c

b
cos
c

a
tan
b

donde a,b, y c, se refieren a las longitudes de los lados y la hipotenusa del triángulo rectángulo
de la Figura 1.3.

En la física, muchas variables, entre ellas, la masa, volumen, energía, temperatura, y el tiempo,
pueden ser completamente descritas por un número único. Les llamamos a estas propiedades
escalares.

Un vector es un objeto matemático usado para caracterizar propiedades que tienen asociadas
magnitud y dirección. Ejemplos de cantidades vectoriales son los desplazamientos (un objeto
es 5 m al este, del origen de coordenadas), la fuerza (estiramos o empujamos un objeto con
una fuerza dada en una particular dirección), y velocidad (un auto viaja a 30 km/s al este).
Representamos a un vector con una flecha. Los vectores son útiles para caracterizar a
cantidades físicas como el desplazamiento, la fuerza, velocidad, aceleración y momentum.
Usamos letras cargadas A,a para designar vectores y letras sin cargar A, a para designar
sus magnitudes, que son escalares.

Definición. El producto de un vector A y un escalar b es un vector cuya magnitud es bA y


cuya dirección es la de A.

Adición y sustracción de Vectores


Supongamos que usted camina 8 pasos al este, para y luego se dirige al norte y camina otros 6
pasos. A pesar de que usted viajo a una distancia total de 14 pasos, su desplazamiento
desde el punto de origen es menor que eso, y no es propiamente al este ni al norte. Aplicando
el teorema de Pitágoras, determinamos la magnitud del desplazamiento

d 82 62 100 10 pasos

Para determinar la dirección del desplazamiento podemos usar una de las funciones
trigonométricas.

9
Instituto Grigory Perelman. Prof. Master Emilio Ortiz
Trepowski. Teléfono 441.320. emiliortiz1@hotmail.com.

1 6
tan 37
8

Definición. Si A y B son dos vectores, el vector C A B es obtenido ubicando al vector


B de manera que su origen coincida con el fin del vector A . El vector C es entonces obtenido
dibujando un vector desde el origen de A al término de B.

Esto se puede extender a tres vectores.

Ejemplo 1.4
Una chica camina 300 m al este. Luego ella camina en una línea recta pero en una dirección
diferente. Al final de su caminata ella está exactamente a 200 m al nor-este de su punto inicial.
¿Qué tan lejos ella se fue luego de que cambió de dirección y en que dirección ella camino en
la segunda vuelta de su viaje?

Sabemos que

R A+B

Por lo tanto,

B R A R A

Usando esta construcción geométrica, una regla, y un protractor, encontramos que B=210 m a
48 a nor-oeste.

Adición de Vectores Usando Componentes Ortogonales


Cualquier vector puede ser visto como la suma de dos o más vectores. En la Figura 1.1.

Cinemática
Entender el movimiento es entender la naturaleza, Leonardo Da Vinci.

La mecánica es el estudio del movimiento de los cuerpos, está convenientemente dividido en


dos partes, cinemática y dinámica. Cinemática es puramente descriptiva y está restringida a
responder a la pregunta: dadas ciertas condiciones iniciales y la aceleración de un objeto en
t 0 y en todos los tiempos subsiguientes, cuál es su posición y su velocidad como funciones
del tiempo? La cinemática no requiere responder a las preguntas de porqué los cuerpos se
aceleran; sólo describe sus comportamientos. La dinámica está preocupada por la causalidad,
con cuales son las causas del movimiento de los objetos.

10
Instituto Grigory Perelman. Prof. Master Emilio Ortiz
Trepowski. Teléfono 441.320. emiliortiz1@hotmail.com.

Movimiento rectilíneo
Los fenómenos naturales tienen lugar en espacios tridimensionales. Sin embargo, antes de
abordar los problemas más complejos de dos y tres dimensiones, nos focalizamos en el más
simple de los casos, el movimiento sobre una línea recta. El análisis de este movimiento, en
particular, el comportamiento de los cuerpos que caen libremente, en uno de los primeros
problemas que ocuparon la atención de los filósofos naturales. Necesitamos conocer el
significado de posición, velocidad y aceleración.

En una dimensión, localizamos un cuerpo especificando su posición coordenada, la cual es la


distancia de un origen de coordenadas arbitrariamente elegido. La unidad de la dimensión
longitud puede ser el metro (m), el centímetro (m), el pie (f) o cualquier unidad conveniente.
La variable coordenada es usualmente designada por x .

Velocidad
Si un objeto se mueva a lo largo del eje de coordenadas, decimos que se está transladando.
Cuando el objeto se mueve, su posición coordenada cambia como el tiempo progresa. Si en el
tiempo t t1 su posición coordenada es x1 , su posición coordenada en el tiempo posterior
t t2 , tendrá un nuevo valor, x2 . Su velocidad media, v , durante el intervalo de tiempo
t t2 t1 es

x2 x1 x
v
t2 t1 t

donde x x2 x1

es su desplazamiento durante el intervalo de tiempo t.

Definición. La velocidad media de un cuerpo es su desplazamiento dividido por el intervalo de


tiempo durante el cual este desplazamiento ocurrió.

Si multiplicamos ambos lados de la ecuación anterior por t , tenemos que

x v t

Ó sea

x2 x1 vt

11
Instituto Grigory Perelman. Prof. Master Emilio Ortiz
Trepowski. Teléfono 441.320. emiliortiz1@hotmail.com.

Por lo tanto, si conocemos la velocidad media, podemos encontrar el desplazamiento del


cuerpo durante el intervalo de tiempo dado; si conocemos su posición inicial, podemos
determinar su posición final.

En la física nosotros hacemos una clara distinción entre velocidad y rapidez (speed). Como
mencionamos antes, la velocidad es una cantidad vectorial; asociamos tanto dirección como
magnitud con la velocidad. En el caso restringido uni-dimensional, la velocidad puede tener
valor positivo o negativo, indicando el translado en la dirección positiva o negativa a lo largo
del eje coordenado. Reservamos el término rapidez para el valor absoluto de la velocidad. Así
un objeto puede tener una velocidad en la dirección x 40m / s ó 40m / s . En cualquier
caso su rapidez es 40m/s.

L
De estas definiciones sigue que las dimensiones de velocidad y rapidez son .
T

Ejemplo 2.1
Un tren está viajando al este a una velocidad de 120 km/h. ¿Qué tan lejos va el tren en 6
segundos?

120km 1h 1000m
120 km/h= 33.3m / s
1h 3600 s 1km

En 6s, el tren cubre una distancia de

x v t 33.3m / s 6 s 200m

Ejemplo 2.2
En el tiempo t 12s un auto se encuentra en x 50m. En t 15s el auto se encuentra en
x 5m. ¿Cuál es la velocidad media y su rapidez media?

5m 50m
v 15m / s
15s 12s

El signo negativo nos dice que el auto se está moviendo en la dirección negativa. Su rapidez es
15 m/s.

Podemos ver, en principio, como determinar la velocidad instantánea. Reducimos el intervalo


de tiempo entre sucesivas observaciones de posición hasta que es infinitesimalmente

12
Instituto Grigory Perelman. Prof. Master Emilio Ortiz
Trepowski. Teléfono 441.320. emiliortiz1@hotmail.com.

pequeño, y a pesar de que x también se aproxima a cero en la medida que hacemos esto, el
x
ratio permanece finito.
t

En términos matemáticos formales expresamos esto como

x
v lim
t 0 t

Si y sólo si la velocidad es constante, podemos usar las expresiones citadas con referencia al
desplazamiento en un intervalo de tiempo dado. Esta condición de velocidad constante se
llama movimiento uniforme. Para el movimiento uniforme, la velocidad instantánea
y promedio son iguales.

Aceleración
En la mayoría de las situaciones de interés, los objetos no mantienen movimiento uniforme
pero sufren cambios en velocidad; esto es, ellos son acelerados.

La definición de aceleración es análoga a la de velocidad.

Definición
La aceleración media durante el intervalo de tiempo t está dado por

v2 v1 v
a
t2 t1 t

Donde v1 y v2 son velocidades instantáneas en los tiempos t1 y t2 .

Multiplicando ambos miembros por t y obtenemos

v a t
v2 v1 a t

Obtenemos la aceleración instantánea por el mismo proceso límite como antes:

v
a lim
t 0 t

Movimiento Unidimensional
13
Instituto Grigory Perelman. Prof. Master Emilio Ortiz
Trepowski. Teléfono 441.320. emiliortiz1@hotmail.com.

Velocidad Media
El análisis del movimiento es un problema fundamental en física, y la forma más simple de
abordarlo es considerar primero los conceptos que intervendrán en la descripción del
movimiento (cinemática), sin considerar todavía el problema de cómo determinar el
movimiento que se produce en una situación física dada (dinámica). Al principio, para
simplificar más la discusión, nos limitaremos a estudiar el movimiento en una sola dimensión –
por ejemplo, el movimiento de un automóvil en línea recta a lo largo de una avenida.

Para describir el movimiento, precisamos primero de una referencia, que en el caso,


unidimensional, es simplemente una recta orientada, en que se escoge el origen 0; la posición
de una partícula en movimiento en el instante t es descrita por la abscisa correspondiente
x t .

Concretamente, podemos pensar en el siguiente ejemplo: x t es la posición en la avenida, en


el momento t , ocupado por el parachoques delantero del automóvil en movimiento a lo largo
de la avenida (en línea recta). Podríamos determinar x t , por ejemplo, filmando el
movimiento del auto y después analizando una a una las imágenes de la película. Sabiendo
cuantas imágenes por segundo son tomadas por el filmador, sabríamos el intervalo de tiempo
t (una fracción de segundo) entre dos imágenes consecutivas de la película, y podríamos así
determinar el valor de x en los instantes: 0, t , 2 t ,…, bastante próximos entre sí
(podríamos también estar filmando simultáneamente un cronómetro fijo en primer plano,
para definir el instante correspondiente a cada imagen).

Otro método para “congelar” una posición instantánea de un objeto en movimiento es tirar
una fotografía de exposición múltiple en que el objeto es iluminado en intervalos de tiempo
t regulares por un “flash” ultra-rápido (estroboscopia). El aspecto de una fotografía de este
tipo para una bolita en caída libre a lo largo de una regla graduada está esquematizada en la
figura de abajo.

Por uno cualquiera de estos métodos, podemos construir una “tabla horaria” del movimiento
del tipo:

t s 0 1 2 3 …

x m 0 0,8 3,1 1,5 …

ó un gráfico, del tipo:

El movimiento más simple es el movimiento uniforme, en el cuál éste gráfico es una recta:
14
Instituto Grigory Perelman. Prof. Master Emilio Ortiz
Trepowski. Teléfono 441.320. emiliortiz1@hotmail.com.

x t a bt (1.1)

Este movimiento se caracteriza por el hecho de que trayectorias iguales x x4 x3 x2 x1


son descritas en intervalos de tiempo iguales t t 4 t3 t2 t1 .

La velocidad v del movimiento uniforme está definida por:

x x t2 x t1
v (1.2)
t t2 t1

o sea, es la razón entre el desplazamiento y el intervalo de tiempo que lleva producirlo.


Gráficamente, v representa el coeficiente angular de la recta (o pendiente) en el gráfico.

m cm km
La velocidad se mide en m s 1
ó ,ó ,…, conforme a las unidades adoptadas.
s s h
Note que v puede tomar tanto valores positivos como negativos. Por la ecuación (1.3) , v 0 ,
cuando x 0 para t 0 , ó sea cuando el movimiento se da en sentido de la
x disminuyendo (marcha atrás en el ejemplo del carro). Podemos también llamar rapidez al
valor absoluto de la velocidad, v .

Si aplicamos (1.4) tomando para t2 un instante t cualquiera y para t1 un instante inicial t0 ,


como:

x t0 x0 posicion inicial (1.5)

entonces obtenemos la “ley horaria” del movimiento rectilíneo uniforme:

x t x0 v t t0 (1.6)

Cualquier movimiento rectilíneo no-uniforme se llama “acelerado”. Podemos extender la


ecuación (1.2) a un movimiento acelerado definiendo vt1 t2 , la velocidad media entre los
instantes t1 y t2 :

x t2 x t1 x
vt1 t2 (1.7)
t2 t1 t

que representa geométricamente, conforme vemos en la Figura, el coeficiente angular


tg de la cuerda que une los extremos 1 y 2 del arco de la curva correspondiente en el

15
Instituto Grigory Perelman. Prof. Master Emilio Ortiz
Trepowski. Teléfono 441.320. emiliortiz1@hotmail.com.

gráfico t , x t . La velocidad media entre t1 y t2 corresponde por lo tanto a la velocidad de

un movimiento uniforme que, partiendo de x t1 en t1 , llega a x t2 en t2 .

Así para un automóvil que recorriese la carretera Sao Paulo-Río (a la que suponemos
rectilínea) en diez horas, la velocidad media entre la partida y la llegada sería de
400km km
40 . Pero esto informa poco sobre el movimiento durante el recorrido. El auto
10h h
pudo haber parado durante algunas horas en algún punto intermedio, y pudo haber
desenvuelto velocidades medias mayores en algunas etapas del recorrido. Sería más
informativo dar el valor de v en diferentes etapas del recorrido, y esto describiría mejor el
movimiento cuanto más cortas sean las etapas, pues el error cometido al aproximar trechos
cortos del recorrido por movimientos uniformes va disminuyendo a medida que acortamos
esos trechos.

Velocidad instantánea
¿Qué significa velocidad en un instante dado t ?

Para ilustrar este concepto, vamos a parafrasear una anécdota utilizada por Feynman en su
curso. Ella tiene la forma de un dialogo entre un estudiante (E) que estaba dirigiendo su auto
de forma a no llegar atrazado al aula de física y el guarda (G) que le hace parar, acusándolo de
exceso de velocidad.

G: Su auto estaba a 120 km/h, y el límite de velocidad aquí es de 60 km/h!

E: ¿Cómo es que yo podía estar a 120 km/h si yo estaba conduciendo aquí hace cerca de 1
minuto, y no durante una hora?

G: Lo que quiero decir es que, si continuase en frente do jeito que estaba, habría recorrido 120
km en una hora.

E. Si hubiese continuando siempre en frente, yo hubiera tenido que romper las leyes de la
física!

G. Bien, eso sería verdadero si hubiese seguido en frente por una hora. Más, si hubiese
120km
continuado en frente por 1 minuto, habría recorrido 2km , y en 1s hubiera recorrido
60
2km
33, 3m , y en 0,1 s hubiera recorrido 3,33 m, y tendría perfectamente dada la situación
60
para proseguir durante 0,1 s.

E. Pero el límite de velocidad es de 60 km/h, y no de 1,66 m en 0,1 s!

16
Instituto Grigory Perelman. Prof. Master Emilio Ortiz
Trepowski. Teléfono 441.320. emiliortiz1@hotmail.com.

G. Es la misma cosa: lo que importa es la velocidad instantánea.

[Observemos la considerable habilidad de G tanto en materia de conocimientos de física como


de paciencia, pero también es preciso reconocer que E tiene también un poco de razón: está
permitido exceder el límite de velocidad en intervalos de tiempo extremadamente cortos,
como en los adelantamientos].

La velocidad de un auto usualmente no sufre ninguna alteración apreciable en intervalos de


tiempo menores a 0,1s, de modo que no es preciso, en este ejemplo, tomar intervalos
menores. Si es necesario, para calcular la velocidad instantánea con precisión cada vez mayor,
podríamos considerar el espacio recorrido en 10 2 s,10 3s,... Cuanto menor t (y en
x
consecuencia también el x correspondiente), más el valor de se aproxima a la velocidad
t
instantánea.

Ejemplo: En la experiencia de caída libre de la bolita, el gráfico t , x t tendría la forma de


m
una parábola x t 2 , donde para x en m y t en s, el valor de 5 ; tomemos:
s2

x t 5t 2 (1.8)

¿Cuál es la velocidad instantánea para t 1s ? Con centro en el instante t 1s , calculemos la


velocidad media, a partir de instantes anteriores y para instantes posteriores, tomando
t 1s, 0,1s, 0,01s,...

x 1 x 0 5 0 m
v0 1 5
1 0 1 0 s
t 1s
x 2 x 1 20 5 m
v1 2 15
2 1 2 1 s

x 1 x 0,9 5 4, 05 m
v 0,9 1 9,5
1 0, 9 1 0,9 s
t 0,1s
x 1,1 x 1 6, 05 5 m
v1 1,1 10,5
1,1 1 1,1 1 s

x 1 x 0,99 5 4,905 m
v 0,99 1 9, 95
1 0,99 1 0,99 s
t 0, 01s
x 1, 01 x 1 5,1005 5 m
v1 1,01 10, 05
1, 01 1 1, 01 1 s

17
Instituto Grigory Perelman. Prof. Master Emilio Ortiz
Trepowski. Teléfono 441.320. emiliortiz1@hotmail.com.

Como la parábola es una curva cóncava para arriba, el coeficiente angular de la cuerda que liga
dos puntos de la curva va aumentando en la medida que subimos en la curva, de forma que la
secuencia de arriba debe representar aproximaciones alternadamente por falta o por exceso
m m m m
de la velocidad instantánea v para t 1s : 5 v 15 ; 9,5 v 10,5 ;
s s s s
m m
9,95 v 10,05 … lo que sugiere cual debe ser el valor de v .
s s

m
v 10 para t 1s
s

Este valor debería ser obtenido como caso límite de la secuencia cuando t 0.
Efectivamente,

2 2 2
x x 1 t x 1 51 t 5 5 1 2 t t 10 t 5 t

2
x 1 t x 1 x 10 t 5 t
v1 1 t 10 5 t 10 cuando t 0.
1 t 1 t t

x
Note que, cuando t 0, también x 0, más el cociente tiende a un valor finito e
t
m
igual a 10 en el ejemplo de arriba.
s

Para una función x t , el límite

x t0 t x t0 x dx
lim lim (1.9)
t 0 t t 0 t t t0 dt t t0

se llama derivada de x en relación a t en el punto t0 . Note que dx y dt son notaciones.

En el ejemplo de arriba de la función parabólica, obtenemos que:

dx m
10 en
dt t 1 s

La notación “lim” (límite) para t 0 que podemos aproximar cuanto más queremos al
resultado exacto tomando t suficientemente pequeño, como hicimos en los cálculos
numéricos más arriba. El límite de arriba no siempre existe para cualquier función de t ;
cuando existe, la función se llama diferenciable en el punto t0 . Generalmente, estaremos
lidiando con funciones diferenciables.
18
Instituto Grigory Perelman. Prof. Master Emilio Ortiz
Trepowski. Teléfono 441.320. emiliortiz1@hotmail.com.

Ejemplo: Calcular la derivada de:

x t at 2 bt c (1.10)

donde a,b y c son constantes, en un punto t cualquiera.

Tenemos que:

2 2
x t t a t t b t t c a t2 2t t t b t t c
2
x x t t x t 2a t a t b t
x
2at a t b
t
x
lim 2at b
t 0 t

Ó sea que

dx
2at b (1.11)
dt

Este ejemplo también ilustra los siguientes resultados inmediatos: la derivada de una
constante es nula; la derivada de una suma es la suma de las derivadas;

Galileo y la caída de los cuerpos


El ejemplo más familiar del movimiento rectilíneo uniformemente acelerado es la caída libre
de un cuerpo suelto en reposo. Este fue uno de los problemas analizados por Galileo en sus
trabajos, que dieron origen a la era moderna de la física.

Los griegos de la época clásica encontraron dificultades impasables en el análisis del


movimiento. Esas dificultades estuvieron relacionadas con la formulación de los conceptos
básicos del cálculo infinitesimal (como los de límite, derivada e integral), que nascieron
precisamente del análisis del problema del movimiento. En el Siglo V a.C., Zenon de Eléia
formuló cuatro célebres paradojas, una de las cuales, “Aquiles y la tortuga”, está directamente
relacionada con este problema: Aquiles apuesta una corrida con una tortuga, y él es 10 veces
más veloz que la tortuga. La tortuga parte antes de él, de modo que está a una distancia
d cuando Aquiles parte. Cuando Aquiles llega a la distancia d , la tortuga habrá recorrido una
d
distancia adicional de , y continuará al frente de Aquiles. Cuando Aquiles haya recorrido
10
d d
, la tortuga habrá recorrido , y así por delante: la conclusión de la paradoja es que
10 100

19
Instituto Grigory Perelman. Prof. Master Emilio Ortiz
Trepowski. Teléfono 441.320. emiliortiz1@hotmail.com.

Aquiles nunca conseguirá alcanzar a la tortuga. La dificultad básica de los griegos estaba en
entender que la suma de una serie infinita de intervalos de tiempo que tienden acero
rápidamente (en progresión geométrica) puede ser finita. [Como ejercicio, suponga que la
m m
tortuga recorre a 10 y Aquiles se desplaza a 1 ; la tortuga parte 15 minutos antes que
s s
Aquiles, del mismo punto inicial. ¿Después de cuánto tiempo y en qué punto Aquiles alcanzará
a la tortuga?].

En la física de Aristóteles (Siglo IV a. C.), la materia era analizada en términos de los “Cuatro
Elementos”: Tierra, Agua, Aire y Fuego, cada uno de los cuáles tenía su “lugar natural”: Agua
(océanos) y Tierra abajo, Aire y Fuego (sol, estrellas) arriba. Un elemento descolocado de su
lugar natural procuraría regresar a él: esto explicaría porque el humo sube, y los cuerpos más
pesados (compuestos de “Tierra”) caen. Según, Aristóteles, cuánto más pesado es un cuerpo,
más deprisa cae: una piedra cae más deprisa que una gota de lluvia. Estas ideas, basadas en
observaciones cualitativas, se transformaron en dogma y predominaron durante cerca de 20
siglos!

Galileo Galilei nasció en Pisa en 1564. Recibió la educación Aristotélica tradicional, y fue
enviado por su padre para estudiar medicina a la Universidad de Pisa. Intertanto, se interesó
más por la matemática y consiguió mudarse para ese campo. Con 21 años, tuvo que dejar la
Universidad por falta de recursos y se fue para Florencia. En Florencia, consiguió rápidamente
establecer tal reputación científica que, a los 26 años, fue nombrado Profesor de Matemáticas
de la Universidad de Pisa. Pasó dos años en Pisa, donde hizo muchos enemigos debido a su
espíritu independiente. Después se mudó a la Universidad de Padua, donde permaneció como
profesor durante 18 años. Fue un gran profesor, llegando a tener 2.000 alumnos en su “aula
magna.”

Fue en Pisa que Galileo procuró verificar experimentalmente si las ideas de Aristóteles eran de
hecho válidas (lo que entonces era una actitud revolucionaria). Entretanto, la célebre historia
sobre la bala de cañón y la bala de fusil que habría dejado caer de lo alto de la Torre de Pisa
para verificar si la bala de cañón realmente caía más rápido parece ser apócrifa. Una
experimentación de este tipo parece haber sido hecha por Simon Stevin, un científico holandés
precursor de Galileo, del cual habría tenido conocimiento.

En Padua, Galileo se volvió defensor de la teoría de Copérnico, conforme veremos más tarde.
Volvió a Toscana en 1610, como Filósofo y Matemático de la Corte, y en 1632 publicó su
“Dialogo sobre los Dos Sistemas Principales del Mundo” defendiendo a Copérnico. Poco
después, se verificó el choque con la Inquisición, que mantuvo a Galileo virtualmente como
prisionero. Fue entonces, ya casi ciego, que él escribió su libro más importante, “Discursos y
Demostraciones Matemáticas sobres Dos Nuevas Ciencias”, contrabandeado para Holanda y
publicado en 1638, cuatro años antes de la muerte de Galileo.

20
Instituto Grigory Perelman. Prof. Master Emilio Ortiz
Trepowski. Teléfono 441.320. emiliortiz1@hotmail.com.

Ambos libros fueron escritos en forma de diálogo entre 3 personajes: Salviati (que representa
a Galileo), Simplicio (defensor de Aristóteles) y Sagredo (el cual representa a un observador
imparcial inteligente). En la primera jornada, Salviati refuta a Aristóteles:

“Aristóteles dice que “una bola de hierro de cien libras, cayendo de 100 cúbitos de altura [un
cúbito equivale a cerca de 45 a 50 cm], llega al suelo antes que una bala de una libra que cayó
de un solo cúbito.” Yo digo que llegan al mismo tiempo. Haciendo la experimentación, usted
puede verificar que el mayor precede al menor por dos dedos, o sea, cuando el mayor ya llegó
al suelo, la otra está a dos dedos de altura; usted no puede querer esconder en esos dos dedos
los noventa y nueve cúbitos de Aristóteles…”

Galileo atribuyó las pequeñas discrepancias de tiempo de caída, en el ejemplo citado, al efecto
de la resistencia del aire, que podía afectar más a un cuerpo más leve, explicando así las
observaciones cualitativas en las que Aristóteles se basaba. Más tarde, con la invención de la
máquina neumática, fue posible verificar que los objetos de peso muy diferentes, de hecho,
caían en el mismo tiempo, cuando se eliminaba la resistencia del aire, haciendo el vacío.

Galileo anuncia la Segunda parte de sus “Discursos” explicando cuál es su propósito.

“Mi objetivo es exponer una ciencia muy nueva que trata de un tema muy antiguo. Tal vez
nada en la naturaleza sea más antiguo que el movimiento, y los libros escritos por filósofos
sobre este tema no son pocos ni poco voluminosos; más aún, descubrí con la experimentación
algunas propiedades de él que merecen ser conocidas y que no fueron observadas ni
demostradas hasta ahora. Fueron hechas algunas observaciones superficiales, como, por
ejemplo, de que el movimiento de un cuerpo pesado en caída libre es continuamente
acelerado, más exactamente la forma en la que esta aceleración ocurre no fue anunciado
hasta ahora…

Fue observado que los proyectiles describen algún tipo de trayectoria curva; más ninguno
mencionó el hecho de que esta trayectoria es una parábola. Conseguí demostrar este y otros
hechos; lo que yo considero más importante , fueron abiertos caminos en esta basta y
excelentísima ciencia. Mi trabajo es apenas el comienzo, caminos y metas serán abiertos por
otras mentes, más agudas que la mía, y se exploraran los secretos más remotos.”

Después de definir y discutir el movimiento uniforme, Galileo pasó a tratar el movimiento


uniformemente acelerado, definido como aquel en el que ocurren incrementos iguales de
velocidad en tiempos iguales. Galileo había pensado primero en una definición en que
incrementos iguales de velocidad correspondían a recorridos iguales, más luego percibió que
no era satisfactoria. Así, fue el primero en definir la aceleración.

Un estudio experimental directo de caída libre sería muy difícil en aquella época, porque los
tiempos de caída en las condiciones usuales son muy cortos. Galileo resolvió esta dificultad
disminuyendo la aceleración, con el auxilio de un plano inclinado. En lugar de medir la

21
Instituto Grigory Perelman. Prof. Master Emilio Ortiz
Trepowski. Teléfono 441.320. emiliortiz1@hotmail.com.

velocidad en función del tiempo, lo que habría sido muy difícil, midió la distancia recorrida por
un objeto descendiendo por un plano inclinado a partir del reposo, mostrando que crece con
el cuadrado del tiempo, lo que conforme él había probado en la discusión anterior, es
característico del movimiento uniformemente acelerado.

Las experimentaciones de Galileo, y muchas otras posteriores, acabaron por establecer como
hecho experimental que el movimiento de caída libre de un cuerpo suelto o lanzado
verticalmente, en la medida que la resistencia del aire pueda ser despreciada, es un
movimiento uniformemente acelerado, y que la aceleración es la misma para todos los
cuerpos (a pesar de que sufren pequeñas variciones de punto a punto de la Tierra). Esta
aceleración de la gravedad es indicada por g y su valor aproximado es:

m
g 9,8 (1.12)
s2

Cinemática en dos y tres dimensiones

Denotemos por s1 y s 2 los vectores de posición de una partícula en tiempo t1 y

Problema 1 Bonjorno.

Un joven recorre los lados de un terreno rectangular de dimensiones 40 m y 80 m.

a) ¿Cuál es la distancia recorrida por el joven en dos vueltas completas?

b) ¿Cuál es la distancia recorrida y el desplazamiento en el recorrido ABC?

Solución.

a) Una vuelta: 40m 80m 40m 80m 240m

Dos vueltas completas 480 m.

b) Distancia recorrida es 80m+40m=120 m

d2 802 402
Desplazamiento es
d 802 40 2 89, 44m 40 5m

Problema 2 Bonjorno.
22
Instituto Grigory Perelman. Prof. Master Emilio Ortiz
Trepowski. Teléfono 441.320. emiliortiz1@hotmail.com.

Una persona sale del punto A y camina pasando por los puntos B,C y D, donde se detiene.
En base a la figura, calcule el desplazamiento y el camino recorrido por la persona en los
trechos:

a) AB

b) ABCD

Solución.

a) Desplazamiento=camino recorrido

d2 302 402
d 30 2 402 2500 50m

b) Camino recorrido

50m 40m 50m 140m

Desplazamiento

100 m

Problema 3 Bonjorno.

Considere un automóvil que recorre una pista circular de 80 m de radio. Determine el


desplazamiento y el espacio recorrido por el automóvil durante:

a) un cuarto de vuelta.

b) Media vuelta.

c) Una vuelta.

Solución.

2 80m
a) Camino recorrido= 40 m
4

Desplazamiento= 802 80 2 113,14m

Problema 4 Bonjorno.
La distancia Tierra-Sol es aproximadamente de 149000000 km. ¿Cuál es el espacio
recorrido en km por la Tierra durante una vuelta en su órbita?

23
Instituto Grigory Perelman. Prof. Master Emilio Ortiz
Trepowski. Teléfono 441.320. emiliortiz1@hotmail.com.

Solución.

Circunferencia= 2 r 2 149.000.000 9.38 108

Problema 5 Bonjorno.
Convierta

a) 90 km/h en m/s

Solución

1000m 1h
90km / h 90 km / h 25m / s
1km 60 60s

b) 15 m/s en km/h

m 1km 3600 s
15m / s 15 54km / h
s 1000m 1h

Problema 6 Bonjorno.
¿Cuál es la velocidad en km/h que un avión debe alcanzar para igualar la velocidad de
propagación del sonido en el aire, suponiendo que ésta sea 330 m/s?

Solución

m m 3600s 1km
330 330 1188 km / h
s s 1h 1000m

Problema 7 Bonjorno.

En el instante t1 2s un automóvil pasa por el punto A de una carretera rectilínea y, en el


instante t2 7 s , pasa por el punto B.

Calcule la velocidad escalar media del automóvil en ese trecho.

Solución.

s2 s1 300m 100m 200m


v 40 m / s
t2 t1 7s 2s 5s
24
Instituto Grigory Perelman. Prof. Master Emilio Ortiz
Trepowski. Teléfono 441.320. emiliortiz1@hotmail.com.

Problema 8 Bonjorno.
Un corredor recorre 100 m en 10 s. Determine su velocidad media en km/h.

Solución.

100m m 1km 3600s


v 10 36 km/h
10s s 1000m 1h

Problema 9 Bonjorno.
Un automovilista recorre tres vueltas de un circuito de 4,5 km de longitud, empleando en cada
vuelta los siguientes tiempos: 10 min, 12 min, 12 m 30 s. Calcule en m/s:

a) la velocidad media del automovilista en cada vuelta;

b) la velocidad media del recorrido total.

Solución.

a)

4, 5km 4,5km 1000m 1min m


v1 7,5
10 min 10 min 1km 60s s

4,5km 4,5km 1000m 1min m


v2 6, 25
12 min 12 min 1km 60s s

4,5km 4,5km 1min 1000m m


v3 6
12, 5min 12,5 min 60s 1km s

4, 5km 3 13,5km 13, 5km 1000m 1min m


b) v 6, 521
10 12 12,5 min 34,5 min 34,5 min 1km 60s s

Problema 10 Bonjorno.

Una estrella se halla a una distancia de 4,5 109 km de la Tierra. Sabiendo que la velocidad
de la luz es de 300000 km/s, ¿cuál es el tiempo que emplea la luz de la estrella para
alcanzar la Tierra?

Solución.

25
Instituto Grigory Perelman. Prof. Master Emilio Ortiz
Trepowski. Teléfono 441.320. emiliortiz1@hotmail.com.

Sabemos que

s
v
t
km 4,5 109 km
300000
s t
4,5 109 km 4, 5 109 km s
t 15000s 1,5 104 s
km 300000 km
300000
s

Problema 11 Bonjorno.
Cierta persona viajaba en un automóvil cuyo velocímetro no funcionaba. Deseando saber
cuál era el valor de la velocidad escalar media del automóvil y sabiendo que los postes de
la red eléctrica dispuestos en la margen de la carretera distan 60 m uno de otro, la persona
comenzó a marcar el tiempo en el instante en que pasó frente a un poste (primer poste) y
constató que transcurrieron 45,6 s hasta el instante en que pasó delante del vigésimo
poste. Determine la velocidad escalar media del automóvil, en km/h, constatada en el
intervalo de tiempo durante el cual la persona se desplazó del 1º al 20º poste.

Solución.

60m 19 m m 1km 3600s km


v 25 90
45, 6s s s 1000m 1h h

Problema 12 Bonjorno.
La distancia de la facultad hasta la zona este de la ciudad es de 24 km. Considerando la
velocidad máxima permitida de 80 km/h, ¿cuántos minutos, como mínimo, una persona
debe emplear en el recorrido en un tránsito completamente libre?

Solución.

s
v
t
km 24km
80
h t
24km 24km h 60 min
t 0,3h 0,3h 18min
km 80 km 1h
80
h

Problema 13 Bonjorno.
26
Instituto Grigory Perelman. Prof. Master Emilio Ortiz
Trepowski. Teléfono 441.320. emiliortiz1@hotmail.com.

Dos ciudades A y B distan 600 km. Un automóvil parte de A a las 8h 15 min y llega a B a las
14 h 32 min 20 s.

a) ¿Cuál es el tiempo que se empleó en el viaje?

b) ¿Cuál es la velocidad escalar media del automóvil durante el viaje? Dé la respuesta en


km/h.

Solución.

a)

1h 1h 1h 1h
14h 32 min 20s 8h 15 min 30s
60 min 3600s 60 min 3600s
14,538888 8, 2583333 6, 28h

b)

600km km
v 95,54
6, 28h h

Problema 14 Bonjorno.
Una línea de ómnibus urbano tiene un trayecto de 25 km. Sabiendo que un ómnibus recorre
ese trayecto en 85 min, calcule su velocidad escalar media en km/h.

Solución

25km 60 min km
v 17, 65
85min 1h h

Problema 15 de Bonjorno.
En un camión cisterna en movimiento, uno de los grifos está mal cerrado y gotea a razón de 2
gotas por segundo. Determine la velocidad del camión, sabiendo que la distancia entre las
marcas sucesivas dejadas por las gotas en el asfalto es de 2,5 m.

Solución.

2, 5 2m 5m m
v 5
1s 1s s

27
Instituto Grigory Perelman. Prof. Master Emilio Ortiz
Trepowski. Teléfono 441.320. emiliortiz1@hotmail.com.

Problema 16 de Bonjorno.
Una escalera mecánica de 6 m de altura y 8 m de base transporta una persona de la base hasta
el techo de la escalera en un intervalo de 20 s.

Determine la velocida media de esa persona.

Solución.

La distancia recorrida está dada por d 82 62 100 10m

10m m
v 0,5
20s s

Problema 17 de Bonjorno.
En una mañana calurosa de un día viernes, la fila única de clientes de un banco tiene una
longitud media de 50 m. En promedio la distancia entre las personas en la fila es de 1,0 m. Los
clientes son atendidos por 3 cajas.

Cada caja lleva cerca de 3 min para atender un cliente. Se pregunta:

a) ¿Cuál es la velocidad (media) de los clientes a lo largo de la fila?

b) ¿Cuánto tiempo un cliente demora en la fila?

c) Si uno de los cajeros se retira por 30 min, ¿cuántos metros la fila aumenta?

Solución.

a)

1m 1m 1min 2 m
v 1, 66 10
1min min 60s s

s
v
t
m 50m
b) 1, 6666 10 2

s t
50m s
t 2
3000s 50 min
1, 6666 10 m

c)

28
Instituto Grigory Perelman. Prof. Master Emilio Ortiz
Trepowski. Teléfono 441.320. emiliortiz1@hotmail.com.

Las tres cajas atendían a una velocidad media de 1m/min. En treinta minutos disminuían la
fila en 30 m. Como hay tres cajas, cada caja disminuía la fila en 10 m. Por lo tanto, si una se
retira la fila aumentará en 10 m.

Problema 18 de Bonjorno.
Un automóvil recorre 80 km a 40 km/h y, luego, 10 km a 20 km/h. Determine la velocidad
media del automóvil durante todo el recorrido.

Solución

El primer tramo lo recorre en dos horas y el segundo en 0,5 horas.

90km km km 2 km 0, 5 km
v 36 40 20 36
2,5h h h 2,5 h 2,5 h

Problema 19 de Bonjorno.
Una patrulla de la policía caminera mide el tiempo que cada vehículo emplea para recorrer
un trecho de 400 m de una carretera. Un automóvil recorre la primera mitad del trecho
con una velocidad de 140 km/h. Siendo 80 km/h la velocidad máxima permitida, ¿cuál
debe ser la mayor velocidad media del automóvil en la segunda mitad del trecho para
evitar la multa?

Solución.

La primera mitad de 200 m lo recorre a 140 km/h. Por lo tanto, lo hace en un tiempo de
1, 42857 10 3 horas. La segunda parte la recorre a una velocidad desconocida pero que
deberá permitirle recorre los 200 m restantes en 0,005 horas menos los 0,0014285 horas
del primer tramo. Por lo tanto la ecuación es:

3
1, 42857 10 3 0, 005 1, 42857 10
v 140 km / h y km / h 80 km / h
0, 005 0, 005
40 y 0, 714286 80
km
y 56
h

29
Instituto Grigory Perelman. Prof. Master Emilio Ortiz
Trepowski. Teléfono 441.320. emiliortiz1@hotmail.com.

Problema 20 Bonjorno
Un niño sale de su casa hacia la escuela, realizando en promedio un paso por segundo. El
tamaño medio de su paso es de 0,5 m y él gasta 5 min en el trayecto. ¿Cuál es la distancia
entre su casa y la escuela?

Solución

1paso 0,5m s s
v
s s t 300s
0,5m
s 300s 150m
s

Movimiento Uniforme (MU)


Considerando una persona que conduce un automóvil de tal forma que la aguja del
velocímetro esté siempre en la misma posición, por ejemplo, 50 km/h, durante ese
tiempo.

En esa condición ella recorrerá 50 km en cada hora, esto es, si en 1 h ella recorre 50 km, en
2 h recorrerá 100 km y así sucesivamente. O sea, recorrerá distancias iguales en intervalos
de tiempo iguales.

Para que esto ocurra, la velocidad escalar instantánea debe ser igual a la velocidad escalar
media en cualquier intervalo de tiempo.

El movimiento descrito en esa situación es denominado movimiento uniforme (MU).

v vm constante 0

Se pueden citar algunos ejemplos: una gota de agua descendiendo por un tubo lleno de
aceite, un ciclista con velocidad constante, el viaje de una nave interplanetaria, la subida o
bajada de una escalera mecánica, etc.

En la práctica, los movimientos no son perfectamente uniformes. Si la trayectoria es


rectilínea, el movimiento se dice que es movimiento rectilíneo y uniforme (MRU).

Función Horaria de las posiciones


El estudio de la cinemática se restringe a la elección de un sistema de referencia y al
registro, en términos matemáticos, de las sucesivas posiciones ocupadas por un cuerpo en
el transcurso del tiempo.

30
Instituto Grigory Perelman. Prof. Master Emilio Ortiz
Trepowski. Teléfono 441.320. emiliortiz1@hotmail.com.

Por tanto, partiendo de la posición actual del cuerpo, en un determinado sistema de


referencia, se puede determinar la posición futura en el mismo sistema de referencia.

Así, dados el aquí y ahora del cuerpo –posición y instantes iniciales- para un determinado
observador, se puede prever el allí y el después del cuerpo en relación al mismo
observador.

Por ello, podemos deducir que:

s s 0 vt

La función horaria de las posiciones de un movimiento uniforme es la expresión


matemática que facilita determinar la posición de un cuerpo durante un intervalo de
tiempo en una determinada trayectoria.

Problemas de Aplicación 1

Un cuerpo se mueve sobre una trayectoria rectilínea obedeciendo a la función


horaria s 40 4t (en unidades del SI). Determine:

a) su posición inicial y su velocidad

b) su posición a los 5 s

c) la variación del espacio entre los instantes 2s y 7s

d) el instante en que el cuerpo pasa por la posición 60 m

Solución

a)

s0 20m
m
v 4
s

b)

t 5s

s 20 4 5 40m

c)

Para t 2, t 7
31
Instituto Grigory Perelman. Prof. Master Emilio Ortiz
Trepowski. Teléfono 441.320. emiliortiz1@hotmail.com.

s 20 4 2 20 4 7 20 m 20m

d)

60= 20 4t

t 10s

Problema de Aplicación 2

Dos móviles parten simultáneamente de dos puntos de una recta, separados por
una distancia de 15 m, recorriéndola en la misma dirección y en sentido contrarios,
con velocidades constantes e iguales a 2m/s y 3m/s.

a) ¿Cuánto tiempo, después de la partida, se produce el encuentro?

b) ¿Cuál es la posición del encuentro?

Solución

a)

Fijando como origen el punto donde se encuentra el móvil A, se tienen las


siguientes funciones horarias:

sA s0 A vAt 0 2t
sB s0 B vBt 15 3t

En el encuentro, se tiene:

sA sB
2t 15 3t
t 3s

c) La posición del encuentro en relación al origen es:

m
2t 2 3s 6m
s

Problema de Aplicación 3

Un tren de 200 m de longitud tiene una velocidad escalar constante de 72 km/h.


Determine el tiempo que él emplea para recorrer un puente de 50 m de longitud.
32
Instituto Grigory Perelman. Prof. Master Emilio Ortiz
Trepowski. Teléfono 441.320. emiliortiz1@hotmail.com.

Solución

km km 1000m 1h m
vA 72 72 20
h h 1km 3600s s

La función horaria de las posiciones para el punto A (parte trasera del tren) en el
inicio del recorrido es:

m
s s0 vt 250m 0 20 t
s
250m 1 s
t 250m 12,5s
m 20 m
20
s

Problema 21 de Bonjorno

Un automóvil se mueve según la función horaria s 50 8t (en unidades del SI).

a) ¿Cuál es la posición inicial y la velocidad del automóvil?

b) ¿Cuál es la posición del automóvil en el instante 20s?

c) ¿En qué instante el automóvil pasa por la posición 650 m?

d) ¿Qué distancia el automóvil recorre durante el 10º segundo?

Solución

a) posición inicial igual a 50 m y velocidad instantánea igual a 8.

b) s 50 8 20 210m

c)

650 50 8t
600
t 75s
8

d)

50 8 9 50 8 10 8m

Problema 22 de Bonjorno
33
Instituto Grigory Perelman. Prof. Master Emilio Ortiz
Trepowski. Teléfono 441.320. emiliortiz1@hotmail.com.

Un cuerpo se mueve según una trayectoria rectilínea, obedeciendo a la función


horaria s 60 10t (en unidades del SI).

Determine:

a) su posición inicial y su velocidad

b) su posición en el instante 3s

c) el instante en que pasa por el origen de las posiciones

d) distancia recorrida en el intervalo de 1s a 10 s

Solución

Problema 23 de Bonjorno

Un cuerpo se mueve sobre la trayectoria rectilínea de la figura, obedeciendo a la


función horaria s 4 2t (en unidades del SI).

a) ¿Cuál es la posición del cuerpo en el instante 5s?

b) Determine el instante en que el cuerpo pasa por el punto A.

Problema 24 de Bonjorno

En una carretera se observan un camión y un jeep, ambos dirigiéndose en el mismo


sentido. Sus velocidades son 54 km/h y 72 km/h, respectivamente.

En el instante t=0, el jeep está a 100 m detrás del camión.

Determine:

a) el instante en que el jeep alcanza el camión

b) el camino recorrido por el jeep hasta alcanzar el camión

Problema 25 de Bonjorno

Dos corredores parten, en sentidos opuestos y en el mismo instante, de los


extremos de una pista rectilínea de 600 m de longitud. Sabiendo que sus
velocidades son iguales a 8,5 m/s y 6,5 m/s, calcule después de cuánto tiempo la
distancia entre ellos es de 450 m.

34
Instituto Grigory Perelman. Prof. Master Emilio Ortiz
Trepowski. Teléfono 441.320. emiliortiz1@hotmail.com.

Problema 26 de Bonjorno

Un batallón de infantería sale del cuartel para una marcha a las 5h de la mañana, a
razón de 5 km/h. Después de una hora y media, un soldado sale del cuartel en un
jeep para llevar una información al comandante de la marcha, a lo largo de la
misma carretera, a 80 km/h. ¿Cuántos minutos empleará el soldado para alcanzar
al batallón?

Problema 27 de Bonjorno

¿Cuánto tiempo emplea un tren de 200 m, a una velocidad de 180 km/h, para
atravesar un túnel de 150 m? Exprese la respuesta en segundos.

Gráficos del movimiento uniforme

La matemática, a través de la teoría de las funciones, proporciona medios para


relacionar las magnitudes respecto al movimiento: posición, velocidad y tiempo.

Se puede también relacionar esas magnitudes gráficamente.

a) Posición en función del tiempo s f t

b) Velocidad en función del tiempo v f t

Aceleración escalar media

En casi todos los movimientos de un cuerpo el valor de la velocidad varía en el transcurso


del tiempo.

Para hacer un estudio más profundo del movimiento de un cuerpo, se necesita saber
cuánto varía la velocidad en cada unidad de tiempo; por ejemplo, en cada segundo.

A la magnitud física responsable de la variación rápida o lenta de la velocidad se denomina


aceleración.

Para estudiar la aceleración, se considera un punto material recorriendo la trayectoria de


la figura de al lado.

35
Instituto Grigory Perelman. Prof. Master Emilio Ortiz
Trepowski. Teléfono 441.320. emiliortiz1@hotmail.com.

am y a

Tipos de movimiento

a) Movimiento acelerado

Es aquel en el cual el módulo de la velocidad aumenta en el transcurso del tiempo. Para


que eso ocurra se debe tener la velocidad y la aceleración con el mismo signo.

v a 0

b) Movimiento retardado

Es aquel en el cual el módulo de la velocidad disminuye en el transcurso del tiempo. En ese


caso, la velocidad y la aceleración deben tener signos contrarios.

v a 0

Como ejemplo un automóvil frenando al aproximarse a una persona.

Movimiento uniformemente variado (MUV)


a) Definición

En la mayoría de los movimientos observables en la naturaleza, la velocidad varía durante


el transcurso del tiempo.

En este caso, al movimiento se denomina movimiento variado. Si en el movimiento de un


cuerpo en intervalos de tiempo iguales éste sufre la misma variación de velocidad escalar,
se dice que posee un movimiento uniformemente variado.

Para que eso ocurra en cualquier intervalo de tiempo, la aceleración escalar media debe
ser constante, diferente de cero e igual a la aceleración escalar instantánea.

am a constante 0

c) Funciones Horarias

1º) Velocidad en función del tiempo v t

36
Instituto Grigory Perelman. Prof. Master Emilio Ortiz
Trepowski. Teléfono 441.320. emiliortiz1@hotmail.com.

v v0
am
t t0
am a cte
v v0 at

2º) Posición en función del tiempo s t

1 2
s s0 v0t at
2

3º) Aceleración en función del tiempo a f t

Un móvil que realiza un movimiento uniformemente variado sufre variaciones de


velocidad iguales en intervalos de tiempo iguales.

Para que eso ocurra, la aceleración del cuerpo debe ser constante y diferente de cero.

a cte 0

Ecuación de Torricelli
Es la ecuación que relaciona la velocidad con el espacio recorrido por el cuerpo en un
movimiento uniformemente variado.

v2 v02 2a s

Ahora seguiremos a Blatt para demostrar estas importantes relaciones de la cinemática. Es


importantísimo hacer las demostraciones porque son ellas las que nos permiten recordar
con facilidad las expresiones matemáticas que dan como resultado.

El gran objetivo de la cinemática es generar relaciones que especifiquen la posición y el


movimiento de objetos bajo condiciones iniciales dadas y también conocida la aceleración.

De la definición de velocidad media, tenemos que:

s s0 vt

Si la aceleración es cero, la velocidad es constante; entonces, y sólo entonces, v v. Si


a 0, la velocidad v cambia con el tiempo. En el caso especial que estamos considerando,

37
Instituto Grigory Perelman. Prof. Master Emilio Ortiz
Trepowski. Teléfono 441.320. emiliortiz1@hotmail.com.

movimiento uniformemente variado o aceleración constante y diferente a cero, a a.


Entonces sabemos que

vt v0 at v0 at

La velocidad cambia linealmente con el tiempo, aumentando o decreciendo de acuerdo al


signo de a. Cuando la velocidad es una función lineal del tiempo, la velocidad media es
sólo la media algebraica de las velocidades inicial y final. Esto es

v0 at v0 at
v v0
2 2

y usando este resultado tenemos que

at
st s0 v0 t
2
1 2
st s0 v0t at
2

Otra relación que es también útil se deriva reescribiendo:

vt v0
t
a

y sustituyendo esta expresión para t en la ecuación anterior:

1 2 2
st s0 vt v0
2a

Gráficos del movimiento uniformemente variado

a) Velocidad en función del tiempo v f t

La función horaria de la velocidad en el movimiento uniformemente variado es una recta


v v0 at.

Se puede tener dos casos:

Aceleración positiva o negativa.

Propiedades

38
Instituto Grigory Perelman. Prof. Master Emilio Ortiz
Trepowski. Teléfono 441.320. emiliortiz1@hotmail.com.

1º) El área limitada por el gráfico representativo y por los ejes coordenados entre los
instantes 0 y t1 es igual al valor numérico del espacio recorrido por el cuerpo entre esos
instantes.

Área del trapecio

2º) La tangente del ángulo representa numéricamente la aceleración

b) Posición en función del tiempo s f t

La función horaria de las posiciones del movimiento uniformemente variado es

1 2
s s0 v0t at .
2

Como esta función es de 2º grado en relación al tiempo, su gráfico representativo es una


parábola.

c) Aceleración en función del tiempo a f t

Se sabe que en el movimiento uniformemente variado la aceleración es constante y


diferente de cero.

Propiedad

En el gráfico de la aceleración el área sombreada, mide numéricamente la variación de la


velocidad entre los instantes t1 y t2 .

Caída de los cuerpos


Cuando se lanza un cuerpo verticalmente hacia arriba se observa que él sube hasta cierta
altura y después cae porque es atraído por la Tierra.

De la misma forma se verifica que un cuerpo cae al ser abandonado de una determinada
altura porque es atraído por la Tierra.

Los cuerpos son atraídos por la Tierra porque en torno a ella hay una región llamada
campo gravitacional que ejerce atracción sobre ellos.

Se denomina caída libre al movimiento de subida o de bajada que los cuerpos realizan en
el vacío, en las proximidades de la superficie de la Tierra.

Se puede también despreciar a la resistencia del aire al movimiento de los cuerpos,


durante la subida o la bajada y, en este caso, se los considera como en caída libre.

39
Instituto Grigory Perelman. Prof. Master Emilio Ortiz
Trepowski. Teléfono 441.320. emiliortiz1@hotmail.com.

Estudiando el movimiento de un cuerpo en caída libre, Galileo Galilei llegó a las siguientes
conclusiones:

1. Las distancias recorridas por un cuerpo en caída libre son proporcionales al


cuadrado de los tiempos utilizados en recorreralas, esto es, la función horaria
de las posiciones es de 2º grado.

2. Todos los cuerpos, independientemente de su masa, forma o tamaño, caen


con aceleración constante e igual.

La aceleración constante de un cuerpo en caída libre es denominada aceleración de la


gravedad y se representa por la letra g.

Conclusión:

Si la aceleración de la gravedad es constante y la función horaria de las posiciones es de 2º


grado, la caída libre es un MRUV y, por tanto, valen todas las funciones y conceptos de ese
movimiento.

La aceleración de la gravedad disminuye con la altura y, al nivel del mar, tiene el valor
aproximado de 9,8 m/s 2 .

A pesar de eso se acostumbra, para efecto de cálculos, considerar g=10 m/s 2 .

La aceleración de la gravedad varía también cuando se pasa del ecuador (g=9,78 m/s 2 ) al polo
(g=9,83 m/s 2 ).

Para estudiar la caída de los cuerpos se consideran dos casos: lanzamiento vertical hacia arriba
y lanzamiento vertical hacia abajo.

a) Lanzamiento vertical hacia arriba

Un cuerpo lanzado verticalmente hacia arriba realiza durante la subida un movimiento


rectilíneo uniformemente retardado, pues el módulo de su velocidad disminuye en el
transcurso del tiempo.

b) Lanzamiento vertical hacia abajo

Un cuerpo lanzado verticalmente hacia abajo realiza un movimiento rectilíneo


uniformemente acelerado, pues el módulo de su velocidad aumenta durante el transcurso
del tiempo.

Cinemática Vectorial
Vector

40
Instituto Grigory Perelman. Prof. Master Emilio Ortiz
Trepowski. Teléfono 441.320. emiliortiz1@hotmail.com.

Es el símbolo matemático utilizado para representar el módulo, la dirección y el sentido de


una magnitud física vectorial.

Se representa un vector por medio de una flecha.

v OP P O

Todo vector tiene tres elementos principales:

1) módulo v v 3u

2) dirección de la recta r

3) sentido de O hacia P

El módulo de un vector es la medida de la flecha que él representa. En la representación


anterior el módulo del vector v es igual a tres unidades de medida.

Operaciones con vectores


a) Adición de dos vectores concurrentes

Dados los vectores a y b se obtiene el vector suma R , tal que R a b.

El método del paralelogramo.

El vector suma R tiene las siguientes características:

Módulo R a 2 b 2 2ab cos

Dirección: de la recta OP

Sentido: de O hacia P

La regla del polígono.

b) Sustracción entre dos vectores

Considerando los vectores a A Oyb B O , que forman entre sí un ángulo .

El vector diferencia v d está dado por:

vd a b A O B O A B

41
Instituto Grigory Perelman. Prof. Master Emilio Ortiz
Trepowski. Teléfono 441.320. emiliortiz1@hotmail.com.

Algebraicamente, el vector v d está dado por

Módulo: vd a 2 b 2 2ab cos

Dirección: de la recta AB

Sentido: de B hacia A

c) Producto de un escalar por un vector

Cuando se multiplica un número a por un vector v1 , se obtiene v 2 , tal que:

Intensidad: v2 a v1

Dirección: la misma de v1

si a 0, el mismo de v1
Sentido:
si a 0, contrario al de v1

Vector Opuesto

Se denomina vector opuesto de un vector al vector - a con las siguientes características:

Igual módulo de a

Igual dirección de a

Sentido contrario al de a

1º) Si dos vectores tienen la misma dirección y el mismo sentido, el vector resultante será:

R a b en módulo.

2º) Si dos vectores tienen la misma dirección y sentido contrarios, el vector resultante será
R a b.

Componentes rectangulares de un vector

Considerando el vector a y los ejes x, y.

Proyectando, perpendicularmente, el vector a sobre los ejes x, y se obtienen sus


componentes rectangulares a x y a y .

42
Instituto Grigory Perelman. Prof. Master Emilio Ortiz
Trepowski. Teléfono 441.320. emiliortiz1@hotmail.com.

De la figura, se tiene:

ax
cos
a
ay
sin
a

Vector posición
Considerando un móvil que describe la trayectoria indicada en la figura en relación al
punto O, origen del sistema cartesiano. Sea P la posición del móvil en un instante t.

Se define como vector posición en el instante considerado el vector r P O.

Vector desplazamiento

Sean P1 y P2 las posiciones del móvil en los instantes t1 y t2 , respectivamente.

Se define como vector desplazamiento, entre los instantes t1 y t2 , el vector r P2 P1 .

Observe que s r.

Velocidad vectorial media


Se define como velocidad vectorial media al cociente del vector desplazamiento r y el
tiempo t empleado en ese desplazamiento.

r
vm
t

r
Módulo: vm
t

Dirección: la misma que r

Sentido: el mismo que r

Velocidad vectorial instantánea


Considere el movimiento de un móvil del punto P1 hacia el punto P2 sobre la trayectoria
curva de la figura.

43
Instituto Grigory Perelman. Prof. Master Emilio Ortiz
Trepowski. Teléfono 441.320. emiliortiz1@hotmail.com.

Cuanto más próximo el punto P2 estuviere al punto P1 , el vector r tiende a ser tangente
a la trayectoria por el punto P1 .

Por tanto, para t tendiendo a cero (el instante t2 es prácticamente igual al instante t1 ) ,
el vector velocidad media es denominado vector velocidad instantánea y se indica por v .

v lim v m
t 0

La dirección del vector velocidad instantánea es siempre tangente a la trayectoria y el


sentido es el del movimiento.

Problema de Aplicación
Un conejo camina 80 m hacia el norte, luego se orienta hacia el este y camina 60 m más.
Sabiendo que en todo el recorrido el conejo emplea 10 s, calcule:

a) el módulo del desplazamiento resultante del conejo;

b) el módulo de las velocidades escalar y vectorial media del conejo.

Solución.

Composición de Movimientos
El movimiento resultante de un cuerpo, al describir una trayectoria cualquiera, está
muchas veces compuesto por más de un movimiento.

Ejemplos:

a) La velocidad de un barco al atravesar un río

b) La velocidad de un avión

Observando la composición de los movimientos, Galileo Galilei concluyó que podía estudiar el
movimiento resultante de un cuerpo analizando separadamente los que lo componen y
enunció el principio de independencia de los movimientos.

Si un cuerpo se encuentra bajo la acción simultánea de varios movimientos, cada uno de ellos
se ejecutan como si los demás no existiesen.

Velocidad Relativa
44
Instituto Grigory Perelman. Prof. Master Emilio Ortiz
Trepowski. Teléfono 441.320. emiliortiz1@hotmail.com.

Consideramos ahora como las observaciones que se realizan en diferentes marcos de


referencias están relacionadas unas con otras. Por ejemplo, consideremos dos trenes
que se aproximan uno a otro, cada uno con una velocidad de 80 km/h con respecto a
la Tierra. Los observadores sobre la Tierra detrás de las vías medirán 80 km/h para las
velocidades de cada tren. Los observadores sobre cualquiera de los trenes (un
diferente marco de referencia) medirán la velocidad a 160 km/h para el otro tren que
se les acerca. Similarmente, cuando un auto está viajando a 90 km/h pasa a un
segundo auto viajando en la misma dirección a 75 km/h, el primer auto tiene una
velocidad relativa al segundo auto de 90 km/h-75 km/h=15 km/h.

Cuando las velocidades están a lo largo de la misma línea, la simple adición o


sustracción es suficiente para obtener la velocidad relativa. Pero si ellas no están en la
misma línea, debemos hacer uso de la adición vectorial. Al especificar la velocidad es
por lo tanto importante mencionar cuál es el marco de referencia.

Cada velocidad está marcada con dos subíndices: el primero se refiere al objeto, el
segundo al marco de referencia en el cual tiene esa velocidad. Por ejemplo,
supongamos que un bote va a cruzar un río al lado opuesto. Sea v BW la velocidad del
bote con respecto al agua. Similarmente, v BS es la velocidad del bote con respecto a la
costa, y vWS es la velocidad del agua con respecto a la costa (esto es la velocidad de la
corriente del río).

Lanzamiento oblicuo
Considerando un cuerpo lanzado oblicuamente, con una velocidad inicial v 0 formando un
ángulo con el eje x.

Despreciando la resistencia del aire, el cuerpo describe una trayectoria parabólica debido a la
atracción de la Tierra.

Se puede estudiar ese movimiento imaginando el lanzamiento oblicuo como resultante de la


composición de dos movimientos: uno en dirección horizontal x y otro en dirección vertical y.

En la dirección horizontal el movimiento del cuerpo es rectilíneo y uniforme con velocidad


igual a v 0 x .

En la dirección vertical el movimiento del cuerpo es rectilíneo uniformemente variado con


velocidad inicial igual a v 0 x y aceleración igual a –g.

Observaciones:

45
Instituto Grigory Perelman. Prof. Master Emilio Ortiz
Trepowski. Teléfono 441.320. emiliortiz1@hotmail.com.

1º) El módulo de la velocidad vertical v y disminuye durante el ascenso y aumenta en el


descenso.

2º) En el punto de altura máxima hmáx el módulo de la velocidad en el movimiento vertical es

cero v y 0 .

3º) A la distancia horizontal entre el punto de lanzamiento y el punto de caída del cuerpo se
denomina alcance xmáx . En este punto y=0.

4º) La posición del cuerpo en un determinado instante está determinada por las coordenadas x
e y.

5º) La velocidad en un determinado instante se obtiene por la suma vectorial de las


velocidades vertical y horizontal, esto es, v v 0 x v y . El vector v es tangente a la trayectoria
en cada instante.

Lanzamiento horizontal
Sea un cuerpo lanzado horizontalmente, con velocidad inicial v 0 , de una altura H en relación al
suelo.

En este caso, el cuerpo describe también una trayectoria parabólica, resultante de la


composición de dos movimientos:

a) un movimiento uniforme en la dirección horizontal

b) un movimiento uniformemente variado en la dirección vertical

El lanzamiento horizontal es un caso particular de lanzamiento oblicuo, en el que el ángulo del


lanzamiento es igual a cero, esto es, 00.

Movimiento Circular

Cinemática del movimiento circular

Supongamos que una pequeña y masiva piedra está constreñida a realizar un movimiento
en un sendero circular alrededor de un centro fijo en 0 y atada a una vara de longitud r .
Figura 6.1. Para especificar el movimiento de la piedra en cualquier momento, podríamos
dar sus coordenadas x e y. Sin embargo, una descripción más conveniente y natural está
46
Instituto Grigory Perelman. Prof. Master Emilio Ortiz
Trepowski. Teléfono 441.320. emiliortiz1@hotmail.com.

dada en términos del ángulo entre el eje y alguna dirección arbitraria, tal como la línea
OA. Si y la longitud de la vara r están dadas, entonces sabremos exactamente donde
encontraremos a la piedra.

La razón por la cual las coordenadas polares, r y , son preferidas es que en el


movimiento circular uno de estos cambia con el tiempo, y lo hace de una manera simple si
el movimiento es uniforme. Por contraste, si la piedra atravieza su sendero circular, las
coordenadas x e y (ambas) están cambiando; ellas están dadas por:

x r cos
y r sin

El radio de este sendero circular es medido en la unidad de medida usual de longitud, el


metro. Dos medidas son comunes para especificar el ángulo. Una, la más familiar, es el
1
grado, definido como de un círculo completo, o revolución completa. Esto es, en la
360
medida que la piedra completa un tránsito circular completo, el ángulo cambia en
360 .

La otra medida de ángulo es el radian.

Definición. Un radián es el ángulo formado por el arco cuya longitud es igual al radio del
círculo.

47
Instituto Grigory Perelman. Prof. Master Emilio Ortiz
Trepowski. Teléfono 441.320. emiliortiz1@hotmail.com.

Esto es, el ángulo , medido en radianes, está dado por el ratio de la longitud de arco al
radio

longitud de arco s
radio r

Dado que la longitud de un círculo completo de radio r es 2 r , la conversión entre


radianes y grados está dada por la condición

2 radianes 360
360
ó 1 radian 57,3
2

La siguiente tabla puede ser útil para mantener la medida de radianes en perspectiva.

48
Instituto Grigory Perelman. Prof. Master Emilio Ortiz
Trepowski. Teléfono 441.320. emiliortiz1@hotmail.com.

360 2 radianes
180 radianes

90 radianes
2
60 radianes
3
45 radianes
4
30 radianes
6

Dado que el ángulo está definido por el ratio de dos longitudes, la longitud de cuerda y
el radio, el ángulo es una variable sin dimensión.

Al igual que con el movimiento lineal, podemos definir una velocidad angular,
designada por (omega en minúscula). La velocidad angular media está dada por

2 1
(1.13)
t2 t1 t

donde 1y 2 denotan valores de en t1 y t2 , y

t 0 t (1.14)

La velocidad angular instantánea es obtenida por el mismo proceso de encontrar


el límite que usamos para llegar a la velocidad instantánea:

lim (1.15)
t 0 t
1
La unidad de la velocidad angular es el radian por segundo. Su dimensión es T .

La velocidad angular no es generalmente constante, pero hay muchas situaciones


importantes en la que es, por ejemplo, un satélite en una órbita circular alrededor de la
tierra. Por el otro lado, la velocidad angular de un auto en una autopista circular en la
medida que el auto acelera, no es constante.

La aceleración angular expresa el cambio de una velocidad angular con el tiempo. La


aceleración angular media está expresada por

49
Instituto Grigory Perelman. Prof. Master Emilio Ortiz
Trepowski. Teléfono 441.320. emiliortiz1@hotmail.com.

2 1
(1.16)
t2 t1

y la aceleración angular instantánea por

lim
t 0 t

La unidad de la aceleración angular es el radián por segundo cuadrado. Su dimensión es


2
T .

Si es constante, obtenemos de estas definiciones

t 0 t
t 0

Problema de Aplicación
Un cuerpo se mueve en una trayectoria circular en el sentido antihorario. En los instantes
3s y 5s sus posiciones son 30 y 120 , respectivamente. Calcule:

a) el ángulo descrito en ese intervalo de tiempo;

b) la velocidad angular media.

Solución.

50
Instituto Grigory Perelman. Prof. Master Emilio Ortiz
Trepowski. Teléfono 441.320. emiliortiz1@hotmail.com.

a) el ángulo descrito en ese intervalo de tiempo está dado por

2 1 120 30 90

Debemos ahora convertir estos noventa grados en radianes, lo que se hace de la siguiente
manera

2 radianes 360
x radianes 90
2 90 2
x radianes rad
360 4 2 2

b) la velocidad angular media viene dada por la siguiente expresión

rad
2 1 120 30 90 2 rad
t t2 t1 5s 3s 2s 2s 4 s

51
Instituto Grigory Perelman. Prof. Master Emilio Ortiz
Trepowski. Teléfono 441.320. emiliortiz1@hotmail.com.

Problema 156

rad
Un móvil realiza un movimiento circular con una velocidad angular media de 10 . Calcule
s
el ángulo descrito en 5s.

Solución

La velocidad angular media está dada por

t
rad
10
s 5s
50 rad

Problema 157

rad
Un cuerpo con movimiento circular tiene una velocidad angular media de . Calcule
2 s
en cuánto tiempo él describe un ángulo de 50 rad.

Solución

t
rad 50 rad
2 s t
50 rad
1 50 rad 2 s
t 100s
rad 1 rad
2 s

Movimiento circular uniforme (MCU)

Se dice que un móvil realiza un movimiento circular uniforme cuando su trayectoria es


circular y el módulo del vector de velocidad permanece constante y es diferente de cero,

v1 v2 v3 c 0

Como ejemplo de movimiento circular uniforme se tienen:

52
Instituto Grigory Perelman. Prof. Master Emilio Ortiz
Trepowski. Teléfono 441.320. emiliortiz1@hotmail.com.

a) el movimiento de los extremos de las manecillas de un reloj;

b) el movimiento de las hélices de un ventilador.

Frecuencia y periodo
Un movimiento es llamado periódico cuando se repite de modo idéntico, en intervalos
iguales de tiempo.

Como ejemplo se tienen los citados en el apartado anterior. Por tanto, el movimiento
circular uniforme es un movimiento periódico, pues a cada vuelta completa el móvil está
siempre con las mismas características (posición, velocidad, etc.).

Se denomina periodo T al tiempo empleado por el móvil para realizar una vuelta
completa.

En una vuelta se tiene:

t T
s 2 r

Luego,

s 2 r 2 r 2 r 2
v T T T
t T v r

Se denomina frecuencia f del movimiento al número de vueltas efectuadas en la unidad


de tiempo.

Por tanto:

tiempo número de vueltas


T 1
1 f
T f 1
1
f
T

La unidad de frecuencia en el Sistema Internacional de Unidades es el inverso del segundo


1
, también llamada hertz, que se indica por Hz.
s

53
Instituto Grigory Perelman. Prof. Master Emilio Ortiz
Trepowski. Teléfono 441.320. emiliortiz1@hotmail.com.

Se puede también indicar la frecuencia en rotaciones por minuto (rpm) 60rpm=1Hz.

Relación entre velocidad escalar y angular


Se considera un móvil describiendo, en el sentido antihorario, la trayectoria circular de la
figura de más abajo.

De la figura se tiene:

s
Sabemos que
r

Por lo tanto, en este caso

54
Instituto Grigory Perelman. Prof. Master Emilio Ortiz
Trepowski. Teléfono 441.320. emiliortiz1@hotmail.com.

s
s r
r
dividiendo ambos lados por t
s r
v r
t t

Siguiendo a Blatt, queda una explicación mucho más sólida. En muchos problemas
prácticos, el movimiento circular es de alguna manera relacionado con el movimiento
lineal. Por ejemplo, la rotación de las ruedas de la bicicleta resulta en su translado. En este
caso, cuánto más rápida es la rotación, más rápida es la translación. Obviamente, hay una
relación entre los dos movimientos, ¿pero cuál es esa relación?

La Figura 6.5 muestra un tubo con un punto en su superficie. En la medida que el tubo da
una revolución completa, el punto P transita un sendero circular de radio r y se mueve
una distancia que es igual a 2 r. La longitud del sendero recorrido por un punto del tubo
es sólo la longitud del arco s , y de la definición del radián, esta longitud está relacionada
con el desplazamiento angular por

s (1.17)

Esta relación simple entre la longitud de cuerda y el ángulo sólo se mantiene si el ángulo
está medido en radianes. La ecuación (1.5) es la razón por la cual la medida del radián es
tan conveniente aquí.

Hemos visto que la distancia tangencial alrededor de un círculo de radio r que


corresponde a una rotación de un ángulo de radianes está dado por s r . Relacione
similares simples conectan la velocidad tangencial de un punto sobre el perímetro de una
rueda con su velocidad angular . Desde la definición de v y , tenemos que

s r
v lim lim r lim r
t 0 t t 0 t t 0 t

Similarmente, la aceleración tangencial t está dada por

v r
t lim lim r lim r
t 0 t t 0 t t 0 t

Estas relaciones simples son válidas sólo si las variables angulares están dadas en radianes.

55
Instituto Grigory Perelman. Prof. Master Emilio Ortiz
Trepowski. Teléfono 441.320. emiliortiz1@hotmail.com.

Problema de Ejemplo

Un tambor de radio 0,4 m empieza desde el reposo en la cima de un plano inclinado a


rodar hacia abajo sin resbalarze (Figura 6-8). El tiempo entre su salida y llegada al punto B,
8 m más abajo en el plano, es 10 s. Encuentre la aceleración angular, la velocidad angular
en B, y el número de revoluciones que el tambor ha hecho al viajar desde A a B, asumiendo
que el tambor procede a una aceleración constante en su camino hacia abajo en el plano.

Solución

Dado que el tiempo transcurrido es 10 s, la velocidad trasnacional promedia es


v 8m 10s 0,8m / s. Dado que

v10 v0 v10
v
2 2

56
Instituto Grigory Perelman. Prof. Master Emilio Ortiz
Trepowski. Teléfono 441.320. emiliortiz1@hotmail.com.

v10 2v 2 0,8m / s 1, 6m / s.

La aceleración tangencial es por lo tanto

v 1, 6m / s
t 0,16m / s 2
t 10s

y la aceleración angular, dada por at r , es entonces

0,16m / s 2
0, 4rad / s 2
0, 4m

Desde la ecuación de relación, la velocidad angular en el punto B es

v10 1, 6m / s
10 4rad / s
r 0, 4m

Por último,

s 8m
20rad
r 0, 4m

20rad
Número de revoluciones 3,18rev
2 rad / rev

Problema de Aplicación
Un cuerpo en MCU efectúa 480 vueltas sobre una circunferencia de 0,5 m de radio en 2
min. Determine:

a) la frecuencia;

b) el periodo;

c) la velocidad escalar del cuerpo.

Solución

a)

57
Instituto Grigory Perelman. Prof. Master Emilio Ortiz
Trepowski. Teléfono 441.320. emiliortiz1@hotmail.com.

número de vueltas tiempo (s)


480 120s
f 1
120 f 480
480 1
f 4 4 Hz
120s s

1 1 1 1 1
b) f T s
T f 4 Hz 4 4
s

2
T

2 2 2 4 radian
c) 8
T 1 s s
s
4
m
v wr 8 0,5 4
s

Problema 158
Un cuerpo efectúa 300 vueltas sobre una circunferencia en 2,5 min.

a) ¿Cuál es el periodo del movimiento?

b) ¿Cuál es la frecuencia, en Hz, del movimiento?

Solución

a) y b) primero hallaremos la frecuencia del movimiento.

número de vueltas tiempo (s)


300 150 s
f 1
300
f= 2 Hz
150s

1 1
T 0, 5s
f 2 Hz

Problema 159

58
Instituto Grigory Perelman. Prof. Master Emilio Ortiz
Trepowski. Teléfono 441.320. emiliortiz1@hotmail.com.

Una rueda completa 150 giros por minuto.

a) ¿Cuál es la frecuencia, el periodo y la velocidad angular de la rueda?

b) ¿Cuál es la velocidad escalar de un punto situado a 12 cm del eje de la rueda?

Solución

a) La frecuencia

número de vueltas tiempo (s)


150 60s
f 1
150
f= Hz 2,5 Hz.
60

1 1
T 0, 4 s
f 2,5

2 2 rad
T 5
0, 4 s

rad
b) v r 5 12cm 60 cm s
s

Problema 160
Un disco gira a 45 rpm. Sabiendo que el diámetro del disco es igual a 16 cm, calcule la
velocidad escalar de un punto de su periferia.

Solución
Primero, convertimos los 45 rpm de frecuencia en Hz.

Sabemos que

60rpm 1Hz
45rpm xHz
x 45 / 60 0, 75Hz.

Por lo tanto, sabemos que:

f 0, 75Hz.

59
Instituto Grigory Perelman. Prof. Master Emilio Ortiz
Trepowski. Teléfono 441.320. emiliortiz1@hotmail.com.

También sabemos que

1 1
T 1,33333s
f 0, 75 Hz

Sabemos que existe una relación entre T y . Esta relación es:

2
T

2
1, 3333

2
1,5 rad s
1,3333

Luego también sabemos que existe una relación entre la velocidad escalar y la velocidad
angular que está dada por

v r 1, 5 8cm 12 cm s

Problema 161
Un disco horizontal de radio r=0,30m gira en torno a su eje con una velocidad angular
5 rad s . Halle la velocidad escalar en un punto de su periferia.

Solución

v r 5 0,30 1,5 m s

Problema 162
Un satélite artificial de 30kg será lanzado y girará en torno a la Tierra en una órbita de
altitud igual a 500 km, con una velocidad lineal de 7,56 km/s. Admitiendo que el diámetro
de la Tierra es de 12.000km ¿cuál será el tiempo, en horas, que él empleará para dar una
vuelta completa?

Solución

Sabemos que

v r

60
Instituto Grigory Perelman. Prof. Master Emilio Ortiz
Trepowski. Teléfono 441.320. emiliortiz1@hotmail.com.

km
7,56 6.500 km
s
3
0, 001163 rad s 1,163 10

2 2
T 1719, 69 s 5400s 1,5h
0, 001163

Problema 163

El motor eléctrico de un ventilador efectúa 720 rpm. Sabiendo que OB 25cm y que
OA 50cm, calcule:

a) la velocidad angular de los puntos A y B;

b) la velocidad escalar de los puntos A y B.

61
Instituto Grigory Perelman. Prof. Master Emilio Ortiz
Trepowski. Teléfono 441.320. emiliortiz1@hotmail.com.

Solución
a) la velocidad angular de los puntos A y B.

Convertimos los 720 rpm a Hz

1Hz 60rpm
xHz 720rpm
x 12 Hz

Sabemos que

62
Instituto Grigory Perelman. Prof. Master Emilio Ortiz
Trepowski. Teléfono 441.320. emiliortiz1@hotmail.com.

2
T

1 2
12
24 rad s

Para ambos puntos A y B.

b)

vA r 24 50 1200cm / s 12m / s
vB r 24 25 600cm / s 6m / s

Problema 164
La velocidad de un automóvil se puede medir fácilmente mediante un dispositivo que
registra el número de rotaciones efectuadas por una de sus ruedas, cuando se conoce su
diámetro. Considere, por ejemplo, un neumático cuyo diámetro es de 0,50m. Sabiendo
que el neumático ejecuta 480 rotaciones en cada minuto, determine la velocidad escalar
del automóvil. Adopte 3,14.

Solución

2
T

1 2
8
16 rad / s
v 16 0, 25 12,56m / s.

Problema 165
Una rueda de 60 cm de radio recorre una trayectoria rectilínea con una velocidad de 86,4
km/h, sin deslizarse.

a) ¿Con qué velocidad angular gira esa rueda?

b) ¿Cuál es la frecuencia de esa rueda?

Solución.

63
Instituto Grigory Perelman. Prof. Master Emilio Ortiz
Trepowski. Teléfono 441.320. emiliortiz1@hotmail.com.

v r
24m / s 0, 6
40rad / s
1 2
f
40 20
f 6, 4 Hz
2

Problema 166
La manecilla del minutero de un reloj mide 50 cm.

a) ¿Cuál es la velocidad angular de la manecilla?

b) Calcule la velocidad lineal de la punta de la manecilla.

Solución

a)

2
T

1..........3600 s
f .........1
1
f Hz
3600
3600 2

rad / s
1800

b)

v 50cm cm / s
1800 36

Problema 167
Una bicicleta parte del reposo y recorre 20 m en 4s con aceleración constante.

a) ¿Cuál es la aceleración de traslación de la bicicleta?

64
Instituto Grigory Perelman. Prof. Master Emilio Ortiz
Trepowski. Teléfono 441.320. emiliortiz1@hotmail.com.

b) Sabiendo que las ruedas de la bicicleta tienen 40 cm de radio, ¿con qué frecuencia
estarán girando al final de ese recorrido?

Solución.

Dinámica del Movimiento Circular Uniforme (Blatt)


Ahora que tenemos las ecuaciones apropiadas de la cinemática, dedicamos nuestra atención a
la dinámica del movimiento circular uniforme. Esto es, queremos encontrar la respuesta a la
pregunta: ¿qué fuerza es necesaria para mantener un cuerpo moviéndose alrededor de un
punto fijo a velocidad constante? Este es el problema central que ocupó a los astrónomos
desde tiempos ancestrales, y su solución fue uno de los logros monumentales de Isaac
Newton. Para Aristóteles y la mayoría de sus sucesores esta cuestión no entrañaba ninguna
dificultad; ellas la desecharon simplemente afirmando que el círculo es la más perfecta de las
figuras geométricas, los senderos circulares son “naturales” para los cuerpos celestes, y no
requería ninguna fuerza.

Pero alguna fuerza externa sí es requerida. Sabemos esto, porque la velocidad de un objeto
que se mueve en un sendero circular está continuamente cambiando a pesar de que su rapidez
puede ser fija (Figura 6.9 a) Un cambio en la velocidad implica una aceleración, y para aceleran
un cuerpo debe experimentar una fuerza neta.

Para determinar esa fuerza, primero debemos conocer la aceleración. La Figura 6.9 b) muestra
la diferencia entre dos vectores de velocidad, v f y v 0 , de la Figura 6.9 a), determinada por la
construcción geométrica descrita en el capítulo 1. La aceleración media en el intervalo
v
temporal t durante el cual este cambio en velocidad ocurrió es entonces a y debe ser
t
en la dirección de v , esto es, a lo largo de la base del triángulo isósceles cuyos lados son v f
y v 0 . Es evidente por la construcción que v es perpendicular a s . Por lo tanto, la
aceleración promedio entre P0 y P1 está dirigida hacia el centro del sendero circular.

La fuerza que resulta en este movimiento debe también apuntar hacia el centro. Esta es la
dirección apropiada para la fuerza; si damos vuelta a un objeto atado a una cuerda de longitud
fija, la cuerda está bajo constante tensión y esta tensión es la fuerza que pone al objeto en
movimiento circular. Sabemos de la experiencia cotidiana que el objeto en movimiento estira
hacia fuera sobre la mano que mantiene la cuerda. Desde la tercera ley de Newton, sigue que

65
Instituto Grigory Perelman. Prof. Master Emilio Ortiz
Trepowski. Teléfono 441.320. emiliortiz1@hotmail.com.

la fuerza que la mano ejerce sobre el objeto via la cuerda debe ser una equivalente fuerza
hacia adentro. Llamamos a esta fuerza dirigida hacia adentro que actúa sobre el objeto fuerza
centrípeta, y la aceleración dirigida hacia adentro del objeto aceleración centrípeta.

Hasta aquí, bien. ¿Pero que tan grande es la fuerza centrípeta y cómo depende de la velocidad
angular y el radio del círculo? La experiencia de cada día da algunas ayudas cualitativas. Si
tomamos una cuerpo pesado y lo atamos a un palo y lo damos vuelta arriba de la cabeza,
encontraremos que cuanto más grande es la velocidad angular, mayor es la fuerza que estira
sobre su mano. También si la longitud del palo es alargada y si se mantiene la velocidad
angular, la fuerza que estira sobre sus manos también se incrementa. Así la fuerza centrípeta
se incrementa con un radio mayor y una mayor velocidad angular.

Para derivar una expresión para la fuerza centrípeta Fc , retornamos a la aceleración


centrípeta, ac . En el tiempo t , el objeto recorre una distancia igual a la longitud de cuerda
s r t (ver Figura 6.9a). En este mismo tiempo, el vector de velocidad ha cambiado
r
de dirección alrededor del mismo ángulo ; esto es, el ángulo entre v f y v 0 en la Figura
6.9 a) es . Ahora si permitimos que t sea más y más pequeña para obtener la aceleración
instantánea, la longitud de cuerda, v de la Figura 6.9 b) se aproxima más y más a la longitud
de arco v . Así en el límite tenemos que

v
ac lim v lim v
t 0 t t 0 t

Podemos reescribir este resultado en dos formas convenientes, usando la relación v r.


Obtenemos que

v2
ac
r
ac r 2

La fuerza centrípeta es ahora dada por la segunda ley de Newton

mv 2 2
Fc mac mr
r

Ejemplo de fuerza centrípeta

Dos masas de 1 kg y 0,5 kg son atadas uno a otra por una cuerda sin masa que pasa a través
del hoyo de una tabla horizontal sin fricción (Figura 6.10). La masa de 1 Kg está suspendida
66
Instituto Grigory Perelman. Prof. Master Emilio Ortiz
Trepowski. Teléfono 441.320. emiliortiz1@hotmail.com.

debajo de la tabla y está en equilibrio cuando la otra masa se mueve en un sendero circular de
20 cm de radio sobre la tabla. ¿Cuál es la velocidad angular de la masa de 0,5 kg alrededor del
hoyo sobre la tabla?

Solución. Dado que la masa menor está en equilibrio, su peso, 1kg 9,8m / s 2 9,8 N , debe
ser igual a la tensión en la cuerda. Esta es también la fuerza centrípeta que la cuerda ejerce
sobre la masa rotativa de 0,5 kg. Por lo tanto,

9,8N=(0,5kg)(0,2m) 2

2
98 rad 2 s 2

Así, 9,9rad / s 1,58rev / s.

Aceleración centrípeta
En el movimiento circular uniforme el vector velocidad es constante en módulo pero es
variable en dirección

Como existe variación del vector velocidad, existe aceleración.

La aceleración a está dada por la expresión:

v v 2 v1
a
t t

Si a tiene la misma dirección y el mismo sentido que v , se concluye que la aceleración


está dirigida hacia el centro de la circunferencia, siendo llamada aceleración centrípeta o
aceleración normal y se indica a cp .

Se demuestra que el módulo de la aceleración centrípeta está dado por

v2
acp
r
2
acp r

Donde:

a) v es la velocidad escalar

b) r es el radio de la trayectoria

67
Instituto Grigory Perelman. Prof. Master Emilio Ortiz
Trepowski. Teléfono 441.320. emiliortiz1@hotmail.com.

La aceleración centrípeta tiene por función variar la dirección del vector velocidad
manteniendo el móvil sobre la circunferencia, produciendo el movimiento circular.

En cada posición del móvil el vector a cp es perpendicular al vector v y dirigido hacia el


centro de la circunferencia.

Problema de aplicación

La luna gira en torno a la Tierra, completando una revolución en 27,3 días. Suponiendo que
su órbita sea circular y tenga un radio de 385000 km, determine la aceleración de la luna
en ese movimiento.

Solución.

T=27,3días=2.358720s=2,36 106 s

r= 385000km=3,95 108 m

Como la órbita se supone circular y el movimiento de la luna es uniforme, se tiene que su


aceleración centrípeta es

2
2 2
acp r r
T
2
2 3,14
acp 3,85 108
6 2
2,36 10
acp 0, 0027m / s 2

Problema 168

¿Cuál es la aceleración centrípeta de una partícula que recorre una circunferencia de 6m


de radio con una velocidad escalar de 30 m/s?

Problema 169

Acoplamiento de Poleas
a) Acoplamiento por correa

Considerando dos poleas acopladas conforme indican las figuras.

68
Instituto Grigory Perelman. Prof. Master Emilio Ortiz
Trepowski. Teléfono 441.320. emiliortiz1@hotmail.com.

Para estos tipos de acoplamiento, se tiene:

RA radio de la polea A;

RB radio de la polea B;

vA velocidad escalar de un punto esférico de la polea A;

vB velocidad escalar de un punto periférico de la polea B.

Suponiendo que la correa es inextensible, todos sus puntos tienen la misma


velocidad escalar.

Admitiendo que no hay deslizamiento, los puntos periféricos de cada polea tienen
la misma velocidad escalar, que es igual a la velocidad escalar de la correa, esto es:

vA vB

b) Acoplamiento con el mismo eje


Considerando dos poleas asociadas conforme indica la figura.

En este caso, los puntos A y B describen el mismo ángulo central , en el mismo


intervalo de tiempo.

Para este tipo de acoplamiento, se tiene:

La velocidad angular de un punto de la polea A es igual a la velocidad angular de


un punto periférico de la polea B, esto es:

A B

Problemas de Aplicación
1. Las poleas indicadas en la figura de al lado tienen radios RA 60cm y
RB 10cm. Sabiendo que f A 20rpm , determine el número de rotaciones de
la polea B.

Solución.

Se debe tener:

69
Instituto Grigory Perelman. Prof. Master Emilio Ortiz
Trepowski. Teléfono 441.320. emiliortiz1@hotmail.com.

vA vB A RA B RB 2 f A RA 2 f B RB

f A RA f B RB
20 60 f B 20
fB 120rpm

Respuesta: 120rpm.

2. Las poleas indicadas en la figura giran coaxialmente.

Sabiendo que RA 20cm, RB 60cm y que la velocidad escalar de un punto


periférico de la polea A es 50cm/s, calcule la velocidad del punto X.

Solución.

Se deb tener:

vA vB 50 vx cm
A B vx 150
RA RB 20 60 s

cm
Respuesta: 150
s

Dinámica
Las Leyes del Movimiento de Newton

La Primera Ley

Cada cuerpo continua en su estado de reposo, o de movimiento uniforme sobre


una línea recta, a menos que sea compelido a cambiar ese estado por fuerzas que
se imponen sobre el.

Aquí Newton introduce el concepto de fuerza. Intuitivamente, sabemos lo que es


una fuerza. Es un empujar o estirar que se impone sobre un objeto. Es cualquier
cosa que causa que un cuerpo cambie su estado de movimiento.

La Segunda Ley

70
Instituto Grigory Perelman. Prof. Master Emilio Ortiz
Trepowski. Teléfono 441.320. emiliortiz1@hotmail.com.

Si una fuerza neta es impuesta sobre un cuerpo, causará una aceleración de ese
cuerpo. Esa aceleración está en la dirección de la fuerza neta, y su magnitud es
proporcional a la magnitud de la fuerza neta e inversamente proporcional sobre la
masa del cuerpo.

De acuerdo con la segunda ley, si un cuerpo particular de masa dada se acelera,


m m
digamos a 2 2 como resultado de aplicar una fuerza F, se acelerará a 4 2 si
s s
aplicamos una fuerza de 2F. Esto es,

a F

Donde el símbolo significa “es proporcional a”.

También, si una fuerza F es aplicada a un objeto de 1 kg de masa y produce una


m
aceleración de 2 2 , la misma fuerza aplicada a 2 kg de masa causará que ella se
s
m
acelere a 1 2 . Esto es,
s

1
a
m

Combinando las dos afirmaciones tenemos que:

F
a
m

A pesar de que la segunda ley de Newton no lo dice explícitamente, claramente


implica que la aceleración depende sólo de la fuerza y la masa. La aceleración no
depende del tipo de fuerza, ya sea gravitacional, eléctrica, mecánica, magnética, o
de cualquier otro origen. Tampoco la aceleración depende de la forma del cuerpo o
de su constitución, ya sea plomo o madera, o de su estado, sólido, líquido o
gaseoso. Consecuentemente, la segunda ley puede ser aplicada al estudio, por
ejemplo, de calcular el fluido de un líquido así como el movimiento de los cuerpos
sólidos debido a fuerzas específicas.

El símbolo de proporcionalidad anterior puede ser reemplazado por un signo de


igualdad suponiendo que nos ponemos de acuerdo en una apropiada unidad de

71
Instituto Grigory Perelman. Prof. Master Emilio Ortiz
Trepowski. Teléfono 441.320. emiliortiz1@hotmail.com.

fuerza. La unidad del SI de la aceleración es el metro por segundo, de la masa es el


kilogramo. Si, entonces, podemos escribir la segunda ley de Newton como:

F ma

pero para que esta expresión sea válida, la unidad de fuerza debe ser aquella que
m
aplicada sobre una masa de 1 kg, imparte una aceleración de 1 2 . Así la unidad
s
m
apropidada es el kilogramo metro por segundo al cuadrado kg 2 . Esta unidad del
s
SI tiene el nombre de newton, abreviado N. Un newton actuando sobre una masa de 1
m
kg proporcionará una aceleración de 1 2 . La dimensión de la fuerza es:
s

M L
F 2
T

La ecuación F ma es una relación entre dos cantidades vectoriales, la fuerza F y la


aceleración a . Pero si dos vectores son iguales, sus respectivos componentes a lo largo
de mutuamente ortogonales ejes deben también ser iguales. Así la ecuación vectorial
es equivalente a las tres ecuaciones algebraicas:

Fx max
Fy ma y
Fz maz

Muchos problemas son resueltos más convenientemente mediante la aplicación de las


leyes de Newton a los componentes ortogonales en forma separada.

Masa y Peso

Sobre la superficie de la tierra y también en el espacio, cada objeto de masa finita


experimenta una fuerza gravitacional hacia el centro de la tierra. A pesar de la que la
fuerza gravitacional es prácticamente nula cuando se compara con otras fuerzas de la
naturaleza como las fuerzas electromagnéticas o las fuerzas nucleares, la fuerza de la
gravedad que actúa sobre los cuerpos ordinarios es substancial, dado que
normalmente estamos considerando masas que contienen 1020 ó más átomos
individuales interactuando con una masiva esfera, la tierra. La fuerza que la tierra
ejerce sobre un objeto de masa dada es el peso del objeto sobre la tierra.

72
Instituto Grigory Perelman. Prof. Master Emilio Ortiz
Trepowski. Teléfono 441.320. emiliortiz1@hotmail.com.

Masa y peso son atributos diferentes. Masa es la propiedad que proporciona a un


cuerpo su inercia, su resistencia a cambiar su estado de movimiento. Para causar un
cambio de movimiento debemos ejercer una fuerza, y la fuerza debe ser mayor cuanto
más grande es la masa. Esto es verdadero sobre la superficie de nuestro planeta o en
el espacio exterior. La masa es una propiedad inherente de un cuerpo particular.

Dado que el peso es una fuerza, la unidad del SI del peso es el newton, no el kilogramo.
Sobre la superficie de la tierra, todos los objetos que no están restringidos se aceleran
m
hacia abajo a g=9,8 2 . Consecuentemente, el peso W, esto es, la fuerza de gravedad
s
que actúa sobre un objeto de masa m es,

W=mg

El peso de un objeto de masa m kg es mg N.

La Tercera Ley

A cada acción corresponde siempre una reacción igual opuesta; o, las acciones mutuas
de dos cuerpos sobre cada uno son siempre iguales, y dirigidas a la parte contraria.

Esta ley establece que cuando un cuerpo ejerce una fuerza sobre el otro, el segundo
ejerce una fuerza igual, dirigida en forma opuesta sobre el primero. La fuerza de acción
ejercida por un cuerpo A sobre B es igual en magnitud pero opuesta en dirección a la
fuerza de reacción ejercida por el cuerpo B sobre el cuerpo A. La inherente simetría del
par acción-reacción impide identificar a una de las fuerzas como acción y a la otra
como reacción. Es una cuestión de preferencia.

Consideremos, por ejemplo, el sistema de una masa M sobre una superficie horizontal
y que es estirada hacia la derecha por una cuerda. Este sistema se muestra en la Figura
3.2 (a), y el par de acción-reacción del sistema se ilustra en la Figura 3.2 (b).
Observemos que en cada caso las dos fuerzas del par son iguales en magnitud y
dirigidas opuestamente, y que ellas actúan sobre cuerpos diferentes.

En la Figura 3.2 (b), los subíndices indican sobre que parte del sistema la fuerza actúa y
que parte del sistema ejerce la fuerza. Así, FSM es la fuerza ejercida sobre la superficie
por la masa, en este caso, el peso de la masa, mg.

Las dos fuerzas FRM y FRH , que actúan sobre los dos extremos de la cuerda, no
constituyen un par de acción y reacción. Ellas actúan sobre el mismo cuerpo, la cuerda,
y más aún, no son necesariamente iguales en magnitud. Si el sistema se está
73
Instituto Grigory Perelman. Prof. Master Emilio Ortiz
Trepowski. Teléfono 441.320. emiliortiz1@hotmail.com.

acelerando hacia la derecha y la cuerda tiene una masa finita M R , entonces de


acuerdo con la segunda ley de Newton, el producto M Ra debe ser igual a la fuerza
neta sobre la longitud de la cuerda; esto es, M Ra FRH FRM 0.

En la Figura 3.2 (a), la cuerda transmite la fuerza ejercida por la mano en un punto en
el espacio a otra locación, el punto en el cual la cuerda está atada a M. La cuerda está
bajo tensión, y la tensión en cualquier punto es la magnitud de dos fuerzas opuestas
que se necesitaráina para mantener las dos partes de la cuerda juntas si la misma se
cortara en ese punto. Una cuerda, en constraste con una roca sólida, puede transmitir
sólo fuerzas sensibles, y solo a lo largo de la dirección de la cuerda. Mediante el paso
de una cuerda ideal (sin masa) sobre una polea ideal (sin fricción y sin masa), la
dirección de esta fuerza sensible puede ser cambiada sin afectar su magnitud.

Observemos que la fuerza de reacción que la superficie ejerce sobre la masa es


perpendicular al plano de la superficie. Una línea que es perpendicular a la superficie
se dice que es normal, y esta fuerza de reacción es a menudo referida como la fuerza
de reacción normal. Una superificie libre de fricción sólo puede ejercer una fuerza
normal.

Aplicaciones de las leyes de Newton

Un objeto está en equilibrio si no se está acelerando; por lo tanto, ninguna fuerza neta
debe actuar sobre ella. Esto no significa que ninguna fuerza puede ser aplicada sobre el
cuerpo. Si varias fuerzas actúan simultáneamente, el equilibrio demanda sólo que la
fuerza neta, esto, el vector suma de varias fuerzas, sea nula. Así, una de las condiciones
del equilibrio es:

Fi 0
i

La otra condición para el equilibrio debe también ser satisfecha. Esto tiene que ver con
el movimiento rotacional que lo estudiaremos más adelante.

Fuerza Resultante

Sea una partícula en la cual están aplicadas varias fuerzas. Ese sistema de fuerzas
puede ser sustituido por una única fuerza, la fuerza resultante, que es capaz de
producir en la partícula el mismo efecto que todas las fuerzas aplicadas.

74
Instituto Grigory Perelman. Prof. Master Emilio Ortiz
Trepowski. Teléfono 441.320. emiliortiz1@hotmail.com.

FR F 1 F 2 ... F N

Problema de Aplicación

Dos fuerzas concurrentes, F 1 y F 2 , de intensidades iguales a 4N y 3N actúan en un


mismo punto material, formando un ángulo entre sí. Determine la intensidad de la
fuerza resultante para los siguientes valores de :

a) 00

b) 600

c) 1800

Solución

Consideremos algunos ejemplos. El primero, concerniente a un objeto suspendido por


una cuerda. Queremos saber la tensión en esa cuerda.

Para resolver este o cualquier otro problema de equilibrio o dinámica necesitamos


saber las fuerzas que actuán sobre el objeto.

Para establecer estas fuerzas con claridad, aislamos al cuerpo; esto es, lo mostramos
sin cuerdas o cualquier otro aditivo. Si los añadidos transmiten fuerzas al cuerpo,
entonces mostramos estas fuerzas mediante el dibujo de los vectores de fuerza.

En nuestro ejemplo, dos fuerzas actúan sobre el cuerpo, como se muestra en la Figura
3.3. Una es la fuerza gravitacional, el peso W . La otra fuerza es la tensión T en la
cuerda de soporte. Estas son las únicas fuerzas que actúan sobre el cuerpo aislado. Por
lo tanto, nuestra ecuación de equilibrio luce así:

W T 0

Y por lo tanto,

W T

75
Instituto Grigory Perelman. Prof. Master Emilio Ortiz
Trepowski. Teléfono 441.320. emiliortiz1@hotmail.com.

Esto es, la tensión en la cuerda que actúa sobre el cuerpo es igual en magnitud al peso
del cuerpo pero en dirección opuesta, apuntando hacia arriba.

Todo esto es muy simple. Pero a veces nos olvidamos que cuando un objeto está sobre
una mesa, hay fuerzas que actúan sobre el, su peso y la fuerza de reacción hacia arriba
de la mesa sobre el objeto. Sin esta última, el objeto caería bajo la fuerza de la
gravedad, como lo hace cuando se remueve la mesa de soporte.

Ejemplo 3.1.
Un problema un poco más complicado es ilustrado en la Figura 3.4 (a), que muestra un
objeto Q de peso desconocido W suspendido de una cuerda añadida a dos otras
cuerdas en A. Una cuerda está pegada a la pared en B, la otra pasa a través de una
polea ideal P a otro peso de 15 N. El sistema está en equilibrio cuando las cuerdas PA y
BA hacen ángulos de 37 0 y 530 , respectivamente, con el horizontal. El problema es
determinar el peso desconocido W.

Solución.
Empezamos por aislar el objeto pero tenemos inmediatamente un dilema. Si aislamos
el objeto Q como en la Figura 3.4 (b), vemos que hay dos fuerzas que actúan sobre el,
W que apunta hacia abajo y TA , la tensión que apunta hacia arriba en la cuerda de
soporte. Podemos concluir que TA W pero como ninguna es conocida no hemos
hecho mucho progreso.

En una situación de esta clase, el cuerpo que debemos ver en aislamiento es el punto
A, la conjunción de las cuerdas que transmiten las fuerzas. Este diagrama es mostrado
en la Figura 3.4 (c).

Ya sabemos desde el ejemplo anterior y desde la tercera ley que la fuerza que actúa
sobre A por la cuerda que soporta a Q es igual a W= mQ g . La fuerza que esta cuerda
ejerce sobre Q es hacia arriba, pero la fuerza que la cuerda ejerce sobre el punto de
soporte A está dirigida opuestamente, o hacia abajo. Dado que 15 N de peso añadido a
la cuerda que pasa por la polea P está en equilibrio, la tensión en esa cuerda debe ser
15 N, como se muestra en la Figura 3.4 (c). Finalmente, hay una tensión TB en la
cuerda AB; su magnitud es desconocida.

Ahora escribimos las ecuaciones para esta situación:

76
Instituto Grigory Perelman. Prof. Master Emilio Ortiz
Trepowski. Teléfono 441.320. emiliortiz1@hotmail.com.

Componentes x: (15 N ) cos 37 0 TB cos 530 0

Componentes y: 15 N sin 37 0 TB sin 530 W 0

La primera de estas ecuaciones puede ser resuelta para TB y da como resultado


TB 20 N . Substituyendo este valor en la segunda ecuación y resolviendo para W,
obtenemos W=25 N.

Repetiremos ahora este problema, resolviéndolo con un conjunto diferente de ejes


ortogonales, x y y , mostrado en la Figura 3.4 (d). Dado que 37 0 530 900 , las dos
cuerdas de soporte residen a lo largo de los ejes x y y . Las ecuaciones de equilibrio
ahora se leen como:

Componentes x : 15 N W sin 370 0

Componentes y : TB W cos 37 0 0

Donde hemos descompuesto a W en sus componentes x y y .

Ejemplo 3.2
Este último ejemplo sobre el equilibrio tiene importantes implicaciones prácticas. Una
larga cuerda es estirada entre dos puntos A y B, como en la Figura 3.5 (a). En cada
extremo, la cuerda es atada a una escala que mide la fuerza que ejerce la cuerda sobre
los soportes. Supongamos que la cuerda es estirada hacia abajo en su punto medio por
una fuerza de 400 N, produciendo una deflección tal que los dos segmentos hacen un
angulo de 5 grados con la línea AB. ¿Cuál es la lectura de la escala?

Solución.

De nuevo aislamos el punto de contacto de las fuerzas. Las ecuaciones de equilibrio


son:

Componentes x: T2 cos 50 T1 cos 50 0

Componentes y: T2 sin 50 T2 sin 50 (400 N ) 0

T1 =2295N

77
Instituto Grigory Perelman. Prof. Master Emilio Ortiz
Trepowski. Teléfono 441.320. emiliortiz1@hotmail.com.

La tensión en la cuerda y por lo tanto la fuerza registrada sobre la cuerda en la escala


es 2295 N.

Ahora nos dedicaremos a la segunda ley e ilustraremos aplicaciones con varios


ejemplos.

Ejemplo 3.3.
Un bloque de masa m 2kg reposa sobre un horizonte ideal, libre de fricción. ¿Cuál es
la aceleración del bloque si una fuerza horizontal de 10 N es aplicada a ella?

Solución.

Comenzamos por aislar el cuerpo. Hay tres fuerzas; el peso W mg ; la fuerza de


reacción de la superficie de soporte R; y la fuerza aplica de 10 N. Dado que el bloque
no se mueve ni hacia arriba ni hacia abajo, R W. La fuerza neta que actúa sobre el
cuerpo es la fuerza horizontal de 10 N.

F ma

Ejemplo 3.4.
Supongamos que en lugar de aplicar una fuerza horizontal, ahora estiramos el bloque
con una fuerza de 10 N con una cuerda que hace un ángulo de 37 0 con el horizontal.
¿Cuál es la aceleración del bloque? ¿Cuál es la magnitud de la fuerza de la reacción
normal de la superficie de soporte sobre el bloque?

Solución.

El diagrama del cuerpo libre es mostrado en la Figura 3.7. Descomponemos las fuerzas
en sus componentes vertical y horizontal y añadimos los componentes para encontrar
la fuerza neta:

Componentes x: F cos 370 10 N cos 370 8N

Componentes y: F sin 37 0 R W 10 N 0, 6 R 2kg 9,8m / s 2 R 13, 6 N

Dado que el bloque permanece en la superficie horizontal, Fy 0 y R=13,6N.


Observemos que esta fuerza de reacción es menor que en el Ejemplo 3.3, en el cual
78
Instituto Grigory Perelman. Prof. Master Emilio Ortiz
Trepowski. Teléfono 441.320. emiliortiz1@hotmail.com.

R=W=19,6N. En este caso, una porción del peso del bloque es soportado por la tensión
en la cuerda.

La aceleración en la dirección x está ahora dada por:

Fx 8N
ax 4m / s 2
m 2kg

Ejemplo 3.5.

Una masa de 2 kg se desliza hacia abajo en un plano inclinado sin fricción que hace un
ángulo de 30 0 con la horizontal. La masa empieza desde el reposo. ¿Cuál es la
velocidad después de que se ha deslizado una distancia de 3m?

Ejercicio de Aplicación
Un pasajero de 80 kg de masa está en un ascensor que baja verticalmente con una
aceleración constante de 2 m/s 2 .

Determine la intensidad de la fuerza que el piso del ascensor ejerce sobre el pasajero.

Solución.

En el pasajero actúan las fuerzas peso y normal. Como él está bajando con la misma
aceleración del ascensor, P N A : aplicando el principio fundamental de la dinámica
en el pasajero, se tiene:

FR P NA ma
800 N A 80 2
NA 640 N

Si fuese colocada una balanza en el piso del ascensor ésta indicaría 640 N.

Influencia de la resistencia del aire


El medio en el cual el cuerpo está inmerso (aire o líquido) ofrece también una
resistencia al desplazamiento.

79
Instituto Grigory Perelman. Prof. Master Emilio Ortiz
Trepowski. Teléfono 441.320. emiliortiz1@hotmail.com.

Un cuerpo abandonado de lo alto de un edificio adquiere movimiento acelerado por la


acción de la fuerza de peso. Otra fuerza que actúa en el cuerpo es la fuerza de
resistencia del aire, que tiene la misma dirección y sentido contrario que la fuerza de
peso.

Esa fuerza de resistencia del aire es variable y depende de la velocidad del cuerpo, de
su forma y de la mayor sección transversal en relación a la dirección del movimiento.

Algunos ejemplos:

1) Para una gota de lluvia cuya velocidad es de 2 m/s, la fuerza de


resistencia del aire es proporcional a esa velocidad.

2) Para cuerpos pequeños, cuya velocidad varía entre 24 m/s y 330 m/s, la
fuerza de resistencia del aire es proporcional al cuadrado de la
velocidad.

3) Un paracaídas tiene la forma semiesférica cóncava (área muy grande)


para aumentar la fuerza de resistencia del aire.

4) Automóviles, aviones y peces tienen forma aerodinámica (cortan el aire


y el agua) y área de sección transversal muy pequeña para disminuir la
fuerza de resistencia del aire o del agua.

Tests de Bonjorno.
1. ¿Cuál es el orden de magnitud del número de segundos contenidos en un mes?

a) 10 3

b) 104

c) 105

d) 10 6

e) 10 7

Solución.

60 60 24 30 2, 6 106

80
Instituto Grigory Perelman. Prof. Master Emilio Ortiz
Trepowski. Teléfono 441.320. emiliortiz1@hotmail.com.

2. El intervalo de tiempo de 2,4 min equivale, en unidades del SI, a:

a) 24 s

b) 124 s

c) 144 s

d) 160 s

e) 240 s

Solución.

60 s
2, 4 min 144s
1min

3. En un edificio de 20 pisos (incluyendo la planta baja) el ascensor emplea 36 s para ir de


la planta baja hasta el 20 0 piso. Una persona en el piso X llama el ascensor, que está
inicialmente en la planta baja, y 39,6 s después de la llamada de la persona alcanza la
planta baja. Si no hubo paradas intermedias, y los tiempos de abrir y cerrar la puerta
del ascensor y de entrada y salida de pasajeros son despreciables, se puede decir que
el piso X es:

a) 9

b) 11

c) 16

d) 18

e) 19

Solución

Segundos por piso

36 s s
1,8
20 p p

s
1,8 x 2 39, 6 s
p
x 11 p

4. La velocidad de 54 km/h corresponde a m/s:


81
Instituto Grigory Perelman. Prof. Master Emilio Ortiz
Trepowski. Teléfono 441.320. emiliortiz1@hotmail.com.

a) 10

b) 15

c) 20

d) 27

e) 54

Solución.

km km 1000m 1h m
54 54 15
h h 1km 3600s s

5. Una persona recorre 4 km corriendo con una velocidad escalar media de 12 km/h. El
tiempo transcurrido es de:

a) 3,0 min

b) 8,0 min

c) 20 min

d) 30 min

e) 33 min

Solución.

s s0 vt
4 0 12t
4 4 60 min
t h 0,3333h h 20 min
12 12 h

6. Hace 500 años que Cristóbal Colón partió de Gomera (Islas Canarias) y llegó a
Guatemala (Islas Bahamas), después de navegar cerca de 3000 millas marinas (5 556
km) durante 33 días. Considerando que un día tiene 86400 s, la velocidad media de la
travesía oceánica, en unidades del SI, fue aproximadamente:

m
a) 2 10 2

m
b) 2 10 1

82
Instituto Grigory Perelman. Prof. Master Emilio Ortiz
Trepowski. Teléfono 441.320. emiliortiz1@hotmail.com.

m
c) 2 100
s

m
c) 2 101
s

m
d) 2 102
s

Solución.

s 5556km km 1000m día


v 168,366364 2 10 1 s 0,19 s
t 33días día km 86400s

7. Un móvil se desplaza durante 10 min con una velocidad constante de 5 m/min y,


después, durante 5 min más con una velocidad constante de 8 m/min. La velocidad
media de ese móvil en m/min, en el intervalo de 15 min, es:

a) 3,5

b) 6,0

c) 6,5

d) 13

e) 15

Solución

s s0 vt
m
s1 5 10 min 50m
min
m
s2 8 5 min 40m
min
90m v15
m
v 6
min

8. Una persona, caminando normalmente, tiene una velocidad del orden de 1 m/s. ¿Qué
distancia, aproximadamente, esa persona recorrerá, caminando durante 15 min?

a) quince metros

b) ciento cincuenta metros


83
Instituto Grigory Perelman. Prof. Master Emilio Ortiz
Trepowski. Teléfono 441.320. emiliortiz1@hotmail.com.

c) un kilómetro

d) diez kilómetros

e) noventa metros

Solución

s s0 vt
m
s 1 900s 900m
s

9. Entre las ciudades A y B, que distan 180 km una de la otra, hay un servicio de
transporte por ómnibus. A cada hora un ómnibus sale de la primera a la segunda
ciudad, transitando con una velocidad constante de 60 km/h. Si se viaja en automóvil
de Ba A, también con una velocidad constante de 60 km/h, habrá cruzamiento con los
ómnibus que transitan en sentido contrario. El intervalo de tiempo entre dos
cruzamientos sucesivos es:

a) 10 min

b) 15 min

c) 30 min

d) 45 min

e) 1 h

Solución

km
180 60 t
h
t 3h

58. La inercia de una partícula de masa m se caracteriza:

I – por la incapacidad de esa partícula de modificar, por sí misma, su estado de reposo o de


movimiento rectilíneo uniforme.

II- por la incapacidad de esa partícula de permanecer en reposo cuando una fuerza
resultante es ejercida sobre ella.

III- por la capacidad de esa partícula de ejercer fuerzas sobre otras partículas.

De las afirmaciones anteriores, ¿cuáles son correctas?

84
Instituto Grigory Perelman. Prof. Master Emilio Ortiz
Trepowski. Teléfono 441.320. emiliortiz1@hotmail.com.

a) Sólo II.

b) Sólo III.

c) Sólo I y II.

d) Sólo I y III.

e) I,II y III.

Solución.

59. Una persona de 80 kg de masa está en el polo Norte de la Tierra donde la aceleración
de la gravedad es supuesta con módulo igual a 10 m/s . La fuerza gravitacional que la
2

persona aplica sobre el planeta Tierra:

a) es prácticamente nula.

b) tiene una intensidad igual a 80 kg.

c) tiene una intensidad igual a 80 N.

d) tiene una intensidad igual a 800 N y está aplicada en el suelo donde la persona pisa.

e) tiene una intensidad igual a 800 N y está aplicada en el centro de gravedad de la Tierra.

Solución.

Adición de Velocidades: Marcos de Referencia


Supongamos que remamos una canoa a una velocidad de 6 km/h encaminándola hacia el
norte a través de una correntada de río que corre hacia el este a una velocidad de 3 km/h.
1km 1
Si el río es de un kilómetro de ancho, nos tomará h 10 min alcanzar la otra
km 6
6
h
costa. Durante ese tiempo, el agua habrá llevado a la canoa río abajo al este a una
km 1
distancia de 3 h 0,5km. Así el desplazamiento de la canoa en relación al
h 6
punto inicial de partida durante ese 10 min está en una dirección dada por

tan 1 0, 5 26, 60 al nor-este

y tiene la magnitud

85
Instituto Grigory Perelman. Prof. Master Emilio Ortiz
Trepowski. Teléfono 441.320. emiliortiz1@hotmail.com.

1km
s 1,12km
cos 26, 60

Dado que el desplazamiento real de la canoa está dada por estos valores, la velocidad
media de la canoa durante estos 10 min es

1,12km
v 6, 72km / h; 26, 60 nor-este
1
h
6

Vemos que la velocidad de la canoa con respecto a la costa es la suma de dos


contribuciones, la velocidad relativa al agua sobre la cual flota y la velocidad del agua en
relación a la costa.

A pesar de que la canoa está viajando en la dirección 26,6 0 , nor-este, la persona remando
lo está haciendo hacia el norte. Más aún, para un segundo observador que se encuentra
en el río flotando en la correntada, la canoa aparece viajando al norte. Es sólo relativo a la
costa que la canoa tiene velocidad nor-este, y la correntada una velocidad al este.

Aceleración Centrípeta
Como se ha visto en el estudio del movimiento circular uniforme, el módulo de la
velocidad v permanece constante mientras que su dirección varía en cada instante debido
a la aceleración centrípeta a cp , dirigida hacia el centro de la curva, cuyo módulo es:

v2
a
r

Donde v es la velocidad escalar o el módulo de v y r es el radio de la trayectoria circular.

Fuerza Centrípeta
Por el principio fundamental de la Dinámica, la aceleración que un cuerpo presenta es
producida por una fuerza que tendrá la misma dirección y el mismo sentido de la
aceleración que causó.

FR ma

En el movimiento circular uniforme, a la fuerza resultante que produce la aceleración


centrípeta se denomina fuerza centrípeta F cp , responsable de que el cuerpo mantenga
una trayectoria circular.

86
Instituto Grigory Perelman. Prof. Master Emilio Ortiz
Trepowski. Teléfono 441.320. emiliortiz1@hotmail.com.

F cp m acp

En módulo:

v2
F cp m
r

A continuación se presentan algunos ejemplos que evidencia las aplicaciones de la fuerza


centrípeta.

La piedra se mantiene en trayectoria circular mientras la cuerda está estirada. Al soltarse la


cuerda, se anula la fuerza centrípeta y la piedra tiende a seguir en línea recta tangente a la
trayectoria cicular.

La luna se mantiene en órbita por la fuerza centrípeta debido a la atracción gravitacional


de la Tierra sobre ella.

Se debe al rozamiento entre los neumáticos y la carretera la fuerza centrípeta necesaria


para conservar el automóvil en una trayectoria circular. Eliminándose ese rozamiento, el
automóvil, por inercia, “sale por la tangente”, no consiguiendo completar la curva.

Problemas de Aplicación
Un vehículo de 1000 kg de masa recorre el trecho de una carretera conforme indica la
figura, con una velocidad constante de 18 km/h. Siendo g=10 m/s 2 , determine la
intensidad de la fuerza normal que la carretera ejerce sobre el vehículo en los puntos A y
B.

Solución.

Gravitación Universal

Leyes de Kepler

Primera Ley
Los planetas describen órbitas elípticas en torno al Sol, ocupando éste uno de sus focos.

Donde:

87
Instituto Grigory Perelman. Prof. Master Emilio Ortiz
Trepowski. Teléfono 441.320. emiliortiz1@hotmail.com.

a semieje mayor (radio medio de la órbita descrita);

b semieje menor.

Al punto A de la órbita del planeta más próximo al Sol se denomina perihelio y al punto B más
distante, afelio.

Segunda Ley

Las áreas barridas por el segmento imaginario que une el centro del Sol al centro del planeta
son proporcionales a los tiempos empleados en barrerlas.

A1 A2
cte
t1 t2

Observaciones:

1) Si A1 A2 t1 t2 .

Como s1 s2 v1 v2 .

Esto quiere decir que los planetas se mueven alrededor del Sol con velocidad
variada.

2) La velocidad de los planetas es mayor cuando ellos están más próximos al Sol,
y menor cuando están más lejos.

3) El movimiento de traslación es variado, esto es:

Tercera Ley
El cuadrado del período de revolución de los planetas es proporcional al cubo de sus distancias
medias del Sol.

T2
k
a3

Observaciones:

88
Instituto Grigory Perelman. Prof. Master Emilio Ortiz
Trepowski. Teléfono 441.320. emiliortiz1@hotmail.com.

1) A través de esta ley se puede concluir que el period de translación de un


planeta aumenta con el aumento del semieje mayor de su trayectoria
alrededor del Sol, esto es, aumenta su año.

Ejemplos: Mercurio, año de 88 días terrestres

Plutón, año de 248 años terrestres

2) Las tres leyes de Kepler son válidas para cualquier cuerpo girando alrededor de
una gran masa central, esto es, los satélites naturales y los artificiales.

3) La constante k sólo depende de la masa del Sol y no del planeta que gira
alrededor de él.

Ley de la gravitación universal


Newton explicó las leyes de los movimientos de los planetas por una hipótesis aplicable a la
universalidad de los casos desde la atracción de los planetas hasta las atracciones moleculares
de los cuerpos.

Estudiando el movimiento de la Luna, él concluyó que la fuerza que la mantiene en órbita es


del mismo tipo que la fuerza que la Tierra ejerce sobre un cuerpo ubicado en sus
proximidades.

A esas fuerzas las denominó gravitacionales y luego enunció la ley de la gravitación universal:

Dos cuerpos se atraen con fuerzas proporcionales a sus masas e inversamente proporcionales
al cuadrado de la distancia entre sus centros.

Esas fuerzas tienen la misma intensidad, la dirección que pasa por el centro de los dos cuerpos
y el sentido contrarios.

Mm
F G
d2

Donde:

1) M y m son las masas;

2) G es la constante de gravitación universal;

3) d es la distancia entre los centros de los dos cuerpos;

4) F es la intensidad de la fuerza gravitacional.

89
Instituto Grigory Perelman. Prof. Master Emilio Ortiz
Trepowski. Teléfono 441.320. emiliortiz1@hotmail.com.

La constante G no depende de los cuerpos ni del medio en que se encuentran ni de la


distancia entre ellos. Depende solamente del sistema de unidades utilizado.

El valor en el SI es:

11 Nm 2
G 6, 67 10
kg 2

Aceleración de la gravedad
Alrededor de la Tierra hay una región denominada campo gravitacional, donde los cuerpos
allí colocados sufren su influencia, que se presenta en forma de una fuerza.

Dentro de ese campo los cuerpos son atraídos hacia la Tierra, sufriendo variaciones de
velocidad, en virtud de la aceleración. Esa aceleración es denominada aceleración de la
gravedad y es representada con la letra “g”.

Se tienen dos casos, que son:

1º caso: Aceleración de la gravedad en la superficie de la Tierra.

Todo punto material de masa m ubicado en la superficie de la Tierra es atraído hacia su


centro.

Esa fuerza atractiva es el peso del cuerpo, que está dada por la expresión:

P m g

Pero, debido a la ley de Newton, la intensidad de la fuerza de atracción gravitacional entre


la Tierra y el cuerpo está dada por:

m M
F G
R2

Donde:

1) M masa de la Tierra;

2) R radio de la Tierra.

No teniendo en cuenta la rotación de la Tierra, la fuerza peso es la fuerza propia de


atracción gravitacional. Luego:

mM
P F mg G
R2

90
Instituto Grigory Perelman. Prof. Master Emilio Ortiz
Trepowski. Teléfono 441.320. emiliortiz1@hotmail.com.

M
gsuperficie G
R2

Esta fórmula es la expresión que permite calcular la aceleración de la gravedad en la


superficie de cualquier planeta.

Se nota que la aceleración de la gravedad en la superficie es independiente de la masa m


del punto material que fue atraído.

2º caso: Aceleración de la gravedad para puntos externos a la Tierra.

Si el punto material está a una altura h de la superficie de la Tierra, se tiene:

M M
g G g externo G 2
d2 R h

Observación:

Los cuerpos flotan dentro de una nave espacial porque la fuerza gravitacional (fuerza peso)
que actúa sobre el cuerpo hace el papel de la fuerza centrípeta que actúa sobre él para
mantenerlo en una trayectoria circular. A la sensación de la ausencia del peso de los
cuerpos se llama ingravidez.

Satélite estacionario
Particularmente interesante para la telecomunicación son los satélites que describen una
órbita circular sobre el plano ecuatorial en 86400 s, es decir, 24 h.

Cuando el satélite tiene órbita circular contenida en el plano ecuatorial y el período de


rotación del satélite es igual al período de la Tierra, el satélite permanece en reposo en
relación a un sistema de referencia fijo en la superficie de la Tierra y es llamado satélite
estacionario.

Ellos deben estar a una altura aproximada de h=35 840 km de la superficie de la Tierra.

Energía
El concepto de energía puede ser considerado intuitivo. No es algo que se puede tocar con
las manos, pero se pueden sentir sus manifestaciones. Ejemplos: se siente calor cuando la
madera se quema; el agua de una cascada mueve las turbinas de una usina hidroeléctrica;
se ve la luz emitida por la llama de una vela, etc.

Para evaluar cuantitativamente la energía, se debe medir la transferencia de energía de un


cuerpo a otro, esto es, la transformación de una forma de energía en otra.
91
Instituto Grigory Perelman. Prof. Master Emilio Ortiz
Trepowski. Teléfono 441.320. emiliortiz1@hotmail.com.

Para medir la cantidad de energía transferida de un cuerpo a otro se introducirá el


concepto de trabajo.

Trabajo de una fuerza


El significado de la palabra trabajo, en Física, es diferente de su significado habitual,
empleado en el lenguaje común. Por ejemplo: un hombre que levanta un cuerpo hasta una
determinada altura realiza un trabajo. En Física, el trabajo que una persona realiza al
sostener un objeto a una cierta altura sin moverse es nulo, pues no hay desplazamiento.

El trabajo en Física, está siempre relacionado a una fuerza y a un desplazamiento. Una


fuerza aplicada a un cuerpo realiza trabajo cuando produce un desplazamiento del cuerpo.

Se tienen dos casos, que se pasan a analizar:

1º caso: La fuerza tiene la misma dirección del desplazamiento

Considerando un punto material que, por la acción de la fuerza F , horizontal y constante,


se mueve de la posición A a la posición B, sufriendo un desplazamiento d.

El trabajo de F en el desplazamiento AB está dado por:

TA, B F d

La unidad de trabajo en unidades del SI es el Nm, llamada joule e indicada por J.

Si la fuerza F tiene el mismo sentido que el desplazamiento, el trabajo se dice motor. Si


tiene sentido contrario, al trabajo se denomina resistente.

Por convención:

Tmotor 0 y Tresistente 0

2º caso: La fuerza no tiene la misma dirección del desplazamiento.

Considerando un punto material que, bajo la acción de la fuerza F , pasa de la posición A a


la posición B sufriendo un desplazamiento d.

Descomponiendo la fuerza F , se tiene:

El trabajo de la componente F y en el desplazamiento d es nulo, pues no hay


desplazamiento en la dirección y. Luego, solamente F x realiza trabajo, dado por:

TA , B TF TFx Fx d
92
Instituto Grigory Perelman. Prof. Master Emilio Ortiz
Trepowski. Teléfono 441.320. emiliortiz1@hotmail.com.

Pero Fx F cos ; por tanto:

TA , B F d cos

Observación:

Si la fuerza F es perpendicular a la dirección del desplazamiento, el trabajo de F es nulo,


pues cos 900 0.

Propiedad:

Se puede calcular el trabajo de una fuerza F , constante, utilizando el gráfico:

A F d A TA, B

El área A es numéricamente igual al módulo del trabajo de la fuerza F en el


desplazamiento de A hacia B.

Esta propiedad es válida cuando la fuerza F es variable y también para cualquier


trayectoria.

Problemas de Aplicación
1. Un punto material se desplaza 10 m por la acción de una fuerza de
50 N de intensidad, como se indica en la figura.

Determine el trabajo realizado por la fuerza F en el desplazamiento AB.

Solución.

TA , B F d cos 50 10 cos 600 250 J

2. Un bloque de 10 kg se mueve en línea recta sobre una mesa lisa, en


posición horizontal, bajo la acción de una fuerza variable que actúa
en la misma dirección del movimiento, conforme muestra el gráfico.

Calcule el trabajo realizado por la fuerza cuando el bloque se desplaza


del origen hasta el punto x 5m.

Solución.

93
Instituto Grigory Perelman. Prof. Master Emilio Ortiz
Trepowski. Teléfono 441.320. emiliortiz1@hotmail.com.

Test de Bonjorno al final de la página 198


1. ¿Cuál es el orden de magnitud del número de segundos contenidos en un mes?

a) 103

b) 104

c) 105

d) 106

e) 107

Solución.

60 60 24 30 2592000 2, 6 106

2. El intervalo de tiempo de 2,4 min equivale, en unidades del Sistema Internacional, a:

a) 24 s

b) 124 s

c) 144 s

d) 160 s

e) 240 s

Solución.

El Sistema Internacional mide el tiempo en segundos. Por lo tanto,

60 s
2, 4 min 2, 4 min 144 s
1 min

3. En un edificio de 20 pisos (incluyendo la planta baja) el ascensor emplea 36 s para ir de


la planta baja hasta el 20 piso. Una persona en el piso X llama el ascensor, que está
inicialmente en la planta baja, y 39,6 s después de la llamada la persona alcanza la
planta baja. Si no hubo paradas intermedias, y los tiempos de abrir y cerrar la puerta
del ascensor y de entrada y salida de pasajeros son despreciables, se puede decir que
el piso X es:

a) 9

94
Instituto Grigory Perelman. Prof. Master Emilio Ortiz
Trepowski. Teléfono 441.320. emiliortiz1@hotmail.com.

b) 11

c) 16

d) 18

e) 19

Solución.

s s0 vt
20 v36
20
v 0,5555
36

Es decir el ascensor recorre 0,5555 pisos por cada segundo.

Digamos que la persona está en el piso x . Como el ascensor tiene que primero subir hasta
este piso y luego bajar, la distancia recorrida es 2x .

2 x 0,5555 39, 6
x 11

4. La velocidad de 54 km/h corresponde, en m/s, a:

a) 10

b) 15

c) 20

d) 27

e) 54

Solución.

km 1000 m 1h m
54 15
h 1 km 60 60 s s

5. Una persona recorre 4 km corriendo con una velocidad escalar media de 12 km/h. El
tiempo transcurrido es de:

a) 3,0 min

b) 8,0 min

95
Instituto Grigory Perelman. Prof. Master Emilio Ortiz
Trepowski. Teléfono 441.320. emiliortiz1@hotmail.com.

c) 20 min

d) 30 min

e) 33 min

Solución.

s s0 vmt
4 12t
60 min
t 0,33 h 0,33 h 19,8 min
1h

6. Hace 500 años que Cristóbal Colón partió de Gomera (Islas Canarias) y llegó a
Guanahani (Islas Bahamas), después de navegar cerca de 3000 millas marinas (5 556
km) durante 33 días. Considerando que un día tiene 86 400 s, la velocidad media de la
travesía oceánica, en unidades del Sistema Internacional, fue aproximadamente:

a) 2 10 2
m/s

b) 2 10 1
m/s

c) 2 10 m/s

d) 2 101 m/s

e) 2 10 2 m/s

Solución.

s s0 vt
5556 v33
km 1 dia 1000 m m m
v 168,36 km/dia 168,36 2 2 10
dia 86400 s 1 km s s

7. Un móvil se desplaza durante 10 min con una velocidad constante de 5 m/min y,


después, durante 5 min más con una velocidad constante de 8 m/min. La velocidad
media en m/min, en el intervalo de 15 min, es:

a) 3,5

b) 6,0

96
Instituto Grigory Perelman. Prof. Master Emilio Ortiz
Trepowski. Teléfono 441.320. emiliortiz1@hotmail.com.

c) 6,5

d) 13

e) 15

Solución.

8. Una persona, caminando normalmente, tiene una velocidad del orden de 1 m/s. ¿Qué
distancia, aproximadamente, esa persona recorrerá, caminando durante 15 min?

a) Quince metros

b) Ciento cincuenta metros

c) Un kilómetro

d) Diez kilómetros

97
Instituto Grigory Perelman. Prof. Master Emilio Ortiz
Trepowski. Teléfono 441.320. emiliortiz1@hotmail.com.

e) Noventa metros

Solución.

m 60 s
s vt 1 15 min 900 m 1 km
s 1m

9. Entre las ciudades A y B, que distan 180 km una de la otra, hay un servicio de
transporte por ómnibus. A cada hora un ómnibus sale de la primera a la segunda
ciudad, transitando con una velocidad constante de 60 km/h. Si se viaja en automóvil
de B a A, también con una velocidad constante de 60 km/h, habrá cruzamiento con los
ómnibus que transitan en sentido contrario. El intervalo de tiempo entre dos
cruzamientos sucesivos es:

a) 10 min

b) 15 min

c) 30 min

d) 45 min

e) 1 h

Solución.

98
Instituto Grigory Perelman. Prof. Master Emilio Ortiz
Trepowski. Teléfono 441.320. emiliortiz1@hotmail.com.

10. Dos móviles, A y B parten de un mismo punto x con velocidades de 20 m/s y 50 m/s,
respectivamente. El móvil A recorre una semicircunferencia, mientras que el móvil B
recorre una trayectoria recta. Sabiendo que la distancia 0x es de 1000 m, para que los
dos móviles lleguen juntos al punto y el intervalo de tiempo entre sus partidas debe
ser de:

a) 95 s

b) 117 s

c) 135 s

d) 157 s

e) 274 s

99
Instituto Grigory Perelman. Prof. Master Emilio Ortiz
Trepowski. Teléfono 441.320. emiliortiz1@hotmail.com.

Solución.

Sabemos que en el movimiento uniforme:

s s0 vt

Dividiendo ambos miembros de esta igualdad por R :

s s0 v
t
R R R
0 t

En el movimiento uniforme, el móvil B se desplazará una distancia de 2000 m en


40 s.

2000 50t
2000
t 40 s
50

2 R
Por su parte, el móvil A se desplazará una distancia de m, la mitad de la
2
circunferencia en 157 s.

s v
t
R R
2 1000
2 20
t
1000 1000
t 157 s

100
Instituto Grigory Perelman. Prof. Master Emilio Ortiz
Trepowski. Teléfono 441.320. emiliortiz1@hotmail.com.

Por lo tanto, para que lleguen juntos a y deben salir con una diferencia de 157 s
menos 40 s. O sea con una diferencia de 117 s.

11. Dos automóviles, A y B, se desplazan sobre una misma carretera, en la misma


dirección y en sentidos opuestos, animados, respectivamente de velocidades
constantes v A 90 km/h y vB 60 km/h . En un determinado instante t0 0 h,
pasan por el mismo punto de referencia. Al final de 15 min, contados a partir del punto
de referencia, la distancia entre los automóviles, en km, será de:

a) 10,0

b) 37,35

c) 42,7

d) 54,8

e) 81,3

Solución.

12. Un objeto se desplaza con movimiento rectilíneo uniforme durante 30 s. La figura


representa el gráfico del espacio en función del tiempo. El espacio del objeto en el
instante t=30 s, en metros, será:

101
Instituto Grigory Perelman. Prof. Master Emilio Ortiz
Trepowski. Teléfono 441.320. emiliortiz1@hotmail.com.

a) 30

b) 35

c) 40

d) 45

e) 50

Solución.

13. Cuando un conductor aumenta la velocidad escalar de su automóvil de 60 km/h a 78


km/h en 10 s, él está comunicando al automóvil una aceleración escalar media, en
m/s2 de:

a) 18

b) 0,2

c) 5

d) 1,8

e) 0,5

Solución.

102
Instituto Grigory Perelman. Prof. Master Emilio Ortiz
Trepowski. Teléfono 441.320. emiliortiz1@hotmail.com.

km
v t2 v t1 78 60 18 h 18 km 1h 1000m m
am 0,5
t2 t1 10 10 s 10 h s 3600s 1km s

14. Un avión parte del reposo y después de 20 s despega con una velocidad de 360 km/h.
Admitiéndose la aceleración constante, ¿cuál es su valor en m/s 2 ?

a) 2

b) 5

c) 10

d) 18

e) 72

Solución.

v 360 km 360 km 1000m h m


am 5
t 20 h s 20 h s km 3600s s2

15. La tabla indica la velocidad instantánea de un objeto en intervalos de 1 s. Las


velocidades instantáneas del objeto a los 3,60 s y 5,80 s son, respectivamente:

a) 15,7 m/s y 20,5 m/s

b) 13,8 m/s y 22,6 m/s

c) 14,5 m/s y 19,5 m/s

d) n.d.a.

Solución.

Entre 3s y 4s, hay una modificación de la velocidad instantánea de 2,3 m/s.

dv
Esto nos da la derivada 2, 3 . Por lo tanto, para hallar la velocidad que le
dt
corresponde a 3,6 s, debemos multiplicar 0,6 por 2,3. Esto nos da 1,38 que es lo
que debemos sumar a 13,1, obteniéndose 14,48 m/s.

Para 5,8 s, el procedimiento es el mismo. Se obtiene en este caso 19,5 m/s.

16. La función horaria del movimiento de una partícula se expresa por s t 2 10t 24 en
unidades del SI. La posición del móvil al cambiar de sentido es:

103
Instituto Grigory Perelman. Prof. Master Emilio Ortiz
Trepowski. Teléfono 441.320. emiliortiz1@hotmail.com.

a) 24 m

b) -25 m

c) 25 m

d) 1 m

e) -1 m

Solución.

El punto de referencia para cambiar de sentido es t=0. Por lo tanto, s=24.

17. Un automóvil que parte del reposo emplea 5 s para recorrer 25 m en movimiento
uniformemente variado. La velocidad final del automóvil es de:

a) 5 m/s

b) 10 m/s

c) 15 m/s

d) 20 m/s

e) 25 m/s

Solución.

v v0 at
1 2
s s0 v0t at
2
1 2
25 a5
2
a 2m / s 2
m
v v0 at 2 5 10
s

18. Un tren posee una velocidad de 108 km/h al pasar por un punto A y, después de
recorrer 125 m, pasa por un punto B con una velocidad de 72 km/h. La distancia
recorrida por el tren hasta detenerse, medida a partir del B, es:

a) 50 m

b) 100 m

104
Instituto Grigory Perelman. Prof. Master Emilio Ortiz
Trepowski. Teléfono 441.320. emiliortiz1@hotmail.com.

c) 225 m

d) 301 m

e) 426 m

Solución.

19. Un tren marcha con una velocidad de 20 m/s cuando el maquinista ve un obstáculo a
50 m de él. La desaceleración mínima que debe ser dada al tren para que no haya
choque es de:

105
Instituto Grigory Perelman. Prof. Master Emilio Ortiz
Trepowski. Teléfono 441.320. emiliortiz1@hotmail.com.

a) 4m/s2

b) 2m/s2

c) 1m/s 2

d) 0,5 m/s 2

e) Cero

Solución.

20. ¿Cuál fue la aceleración escalar media del cuerpo entre los instantes t=0 s y t=8 s, en
cm/s ?
2

a) 0,75

b) 1,1

c) 1,5

d) 2

e) 3,2

Solución.

106
Instituto Grigory Perelman. Prof. Master Emilio Ortiz
Trepowski. Teléfono 441.320. emiliortiz1@hotmail.com.

12 0
am 1, 5cm/s 2
8

21. ¿Cuál es la distancia recorrida por el cuerpo entre los instantes t=0s y t=8s, en cm?

a) 8,0

b) 12

c) 24

d) 48

e) 96

Solución.

1 2
s s0 v0t at
2
1
s 0 0 1,5 82 48cm
2

22. Una partícula describe un movimiento representado por el gráfico. Sabiendo que su
posición inicial es de 100 m, su posición final es de:

a) 1 290 m

b) 1 390 m

c) -3 810 m

d) 89 000 cm

e) -38 100 dm

Solución.

107
Instituto Grigory Perelman. Prof. Master Emilio Ortiz
Trepowski. Teléfono 441.320. emiliortiz1@hotmail.com.

23. Un móvil se mueve sobre una recta, con su posición variando con el tiempo como
indica el gráfico. La aceleración de ese movimiento es:

a) 2m/s2

b) 3m/s 2

c) 4 m/s 2

d) 5 m/s 2

e) 6 m/s 2

Solución.

108
Instituto Grigory Perelman. Prof. Master Emilio Ortiz
Trepowski. Teléfono 441.320. emiliortiz1@hotmail.com.

24. Un móvil que parte del reposo ejecuta un movimiento rectilíneo cuya aceleración
escalar varía con el tiempo conforme al diagrama. Se puede afirmar que, al final de 4s,
el espacio recorrido es:

a) 45 m

b) 100 m

c) 180 m

d) 30 m

e) 50 m

Solución.

109
Instituto Grigory Perelman. Prof. Master Emilio Ortiz
Trepowski. Teléfono 441.320. emiliortiz1@hotmail.com.

25. Un cuerpo es lanzado del suelo hacia arriba, con una velocidad inicial de 100 m/s.
Siendo g 10m/s 2 . ¿cuál es la altura en que se detiene y el tiempo que emplea en
caer?

a) 1 000 m, 100 s

b) 750 m, 50 s

c) 500 m, 25 s

d) 500 m, 10 s

e) 350 m, 5 s

Solución.

v v0 at
0 100 10t
t 10s

Le lleva 10 s llega a la altura máxima. Luego la altura máxima estará dada por:

1 2 1
s s0 v0t at 100 10 10 10 2 500m
2 2

Al caer:

110
Instituto Grigory Perelman. Prof. Master Emilio Ortiz
Trepowski. Teléfono 441.320. emiliortiz1@hotmail.com.

1 2
500 0 0 t 10t
2
100 t 2
t 10s

26. Lanzando una piedra verticalmente hacia arriba y considerando la aceleración de la


gravedad y la resistencia del aire, se puede afirmar que:

a) el tiempo de subida es menor que el tiempo de bajada

b) el tiempo de subida es mayor que el tiempo de bajada

c) el tiempo de subida es igual al de bajada

d) la fuerza de la resistencia del aire actúa en el mismo sentido de la gravedad tanto


en la subida como en la bajada

e) Ninguna de las afirmaciones es verdadera

Solución.

Como puede observarse en el ejercicio anterior, el tiempo de subida es igual al tiempo de


bajada.

27. Un cuerpo es abandonado a partir del reposo y alcanza el suelo con una velocidad de
20 m/s. Considerando g=10 m/s 2 , el cuerpo cae de una altura de:

a) 200 m

b) 100 m

c) 50 m

d) 20 m

e) 10 m

Solución.

v v0 at
20 0 10t
t 2s

111
Instituto Grigory Perelman. Prof. Master Emilio Ortiz
Trepowski. Teléfono 441.320. emiliortiz1@hotmail.com.

1 2
s s0 v0t at
2
1
s 0 0 10 4
2
s 20m

28. Una piedra, que parte del reposo, cae de una altura de 20 m. Se desprecia la
resistencia del aire y adopta g=10 m/s 2 . La velocidad de la piedra al alcanzar el suelo y
el tiempo empleado en la caída, respectivamente, valen:

a) v=20 m/s y t=2 s

b) v=20 m/s y t=4 s

c) v=10 m/s y t=2 s

d) v=10 m/s y t=4s

Solución

1 2
20 0 10t
2
t 2s
m
v 10 2 20
s

29. Un cuerpo es abandonado en caída libre de lo alto de un edificio. Suponiendo la


aceleración de la gravedad constante, de módulo g=10 m/s 2 y despreciando la
resistencia del aire, la distancia recorrida por el cuerpo durante el quinto segundo es:

a) 125 m

b) 80 cm

c) 205 m

d) 5 m

e) 45 m

Solución.

112
Instituto Grigory Perelman. Prof. Master Emilio Ortiz
Trepowski. Teléfono 441.320. emiliortiz1@hotmail.com.

118. Dos bloques en posición vertical están unidos por una cuerda. Otra cuerda es amarrada al
bloque superior. La fuerza F necesaria para mantener el sistema en reposo vale, en kgf:

a) 2

b) 3

c) 4

d) 6

e) 20

Estática
“Denme un punto de apoyo y moveré el mundo.”

La Estática es de interés relevante en el cálculo de grandes estructuras, tales como


puentes y edificios, donde la ausencia de rotación y traslación son condiciones
necesarias y suficientes para mantenerlos en equilibrio.

Esta ciencia ilustra y justifica la diferencia entre intentar cerrar una puerta
aplicando la fuerza próxima a las bisagras o distante de ellas, o aplicar la fuerza al
punto más distante o más próximo al eje de rotación de una llave de boca, cuando
se desea retirar la tuerca de un tornillo.

Principio de traslación de fuerzas


Si se estira un cuerpo de forma cúbica con una fuerza F en el punto A, centro de
la cara lateral derecha del cuerpo,

Se puede también empujar ese cuerpo, pasando el punto de aplicación de la


fuerza F hacia el punto B, centro de la cara lateral izquierda.

113
Instituto Grigory Perelman. Prof. Master Emilio Ortiz
Trepowski. Teléfono 441.320. emiliortiz1@hotmail.com.

En los dos caso, el efecto de la fuerza F sobre el cuerpo es el mismo.

Esto siempre ocurrirá si el punto de aplicación de la fuerza sobre el cuerpo


estuviera en la línea de acción de esa fuerza.

El efecto de la fuerza F sobre el cuerpo es el mismo si ella es aplicada a los puntos


A, B ó C de su línea de acción.

El efecto de una fuerza sobre un cuerpo no se altera cuando se


desplaza su punto de aplicación a lo largo de su línea de acción.

Centro de Gravedad
Se puede considerar los cuerpos materiales como un sistema de partículas, cada
una de las cuales es atraída por la Tierra con una fuerza igual al peso de la
partícula.

La resultante total de todas esas fuerzas parciales es el peso total del cuerpo. Sea
G el punto en el cual se puede considerar que se aplica el peso total de ese cuerpo.
El punto G se denomina centro de gravedad del cuerpo.

114
Instituto Grigory Perelman. Prof. Master Emilio Ortiz
Trepowski. Teléfono 441.320. emiliortiz1@hotmail.com.

Centro de gravedad de un cuerpo es el punto de aplicación de


la fuerza peso.
La Tierra atrae al cuerpo como si toda su masa estuviese localizada en el centro de
gravedad.

Par a cuerpos homogéneos, esto es, de masa uniformemente distribuida, que


admiten un eje de simetría, sus centros de gravedad están sobre ese eje.

En un cuerpo homogéneo de forma cúbica, el centro de gravedad es el punto de


intersección de las diagonales.

Problemas de Aplicación
Determine las coordenadas del centro de gravedad de los cuerpos homogéneos y
de espesores despreciables que se indican en las figuras.

Solución.

Estableciendo los ejes x e y, se tiene:

115
Instituto Grigory Perelman. Prof. Master Emilio Ortiz
Trepowski. Teléfono 441.320. emiliortiz1@hotmail.com.

Problemas Propuestos
373. Halle las coordenadas del centro de gravedad de los cuerpos homogéneos,
cuyos espesores son despreciables, que se indican en las figuras.

Solución

116
Instituto Grigory Perelman. Prof. Master Emilio Ortiz
Trepowski. Teléfono 441.320. emiliortiz1@hotmail.com.

374. Calcule las coordenadas del centro de gravedad de la chapa triangular


equilátera y de espesor despreciable que se indica en la figura. El lado del triángulo
mide 40 3 cm.

Solución

Equilibrio estático de un punto material

117
Instituto Grigory Perelman. Prof. Master Emilio Ortiz
Trepowski. Teléfono 441.320. emiliortiz1@hotmail.com.

Para que un punto material se encuentre en equilibrio estático es necesario y


suficiente que la resultante de todas las fuerzas que en él actúan sea nula.

Punto material de equilibrio R 0

Problema de aplicación
Un cuerpo de 80 N de peso se mantiene en equilibrio mediante hilos ideales,
conforme indica la figura.

Determine las intensidades de las tracciones soportadas por los hilos AB y AC.

Solución.

Descomponiendo las fuerzas en sus componentes, en los ejes x e y, se tiene:

118
Instituto Grigory Perelman. Prof. Master Emilio Ortiz
Trepowski. Teléfono 441.320. emiliortiz1@hotmail.com.

Problemas Propuestos

119
Instituto Grigory Perelman. Prof. Master Emilio Ortiz
Trepowski. Teléfono 441.320. emiliortiz1@hotmail.com.

375. El esquema representa dos cuerpos X e Y de pesos iguales a 30 N y 20 N,


respectivamente, suspendidos por hilos, 1 y 2, ideales.

Calcule la intensidad de la tracción en los hilos 1 y 2.

Solución

376. Un cuerpo de 400 N de peso se halla en equilibrio, como se ilustra en la figura. Determine
la intensidad de las fuerzas tensoras en las cuerdas, supuestas de pesos despreciables.

Solución

120
Instituto Grigory Perelman. Prof. Master Emilio Ortiz
Trepowski. Teléfono 441.320. emiliortiz1@hotmail.com.

377. Un bloque de peso P 80 N está suspendido por hilos, como indica la figura.

a) ¿Cuál es la intensidad de la tracción que se ejerce en el hilo AB?

b) ¿Cuál es la intensidad de la tracción que se ejerce en el hilo horizontal ABC?

378. En un gimnasio hay dos puntos fijos, A y B, de los cuales se suspende una lámpara de 600
N de peso, mediante los hilos AC y BC, como se indica en la figura.

Determine la intensidad de la fuerza de tracción en cada hilo.

379. Calcule la fuerza en cada uno de los hilos flexibles AB y AC, sabiendo que el peso del
cuerpo colgado es de 26 N. Considere sen45 cos 45 0, 7 y cos 30 0,8.

380. La posición de una lámpara se puede ajustar con la ayuda de un contrapeso, como se
indica en el esquema.

Para la situación representada, el contrapeso es de 10 N. ¿Cuál es el peso de la lámpara?


121
Instituto Grigory Perelman. Prof. Master Emilio Ortiz
Trepowski. Teléfono 441.320. emiliortiz1@hotmail.com.

381. El hombre que se indica en la figura posee 70 kg de masa y se halla en equilibrio.


Sabiendo que él se halla en una posición horizontal, que su peso actúa en el punto A y que
cos 30 0,8, calcule:

a) la fuerza tensora en el hilo;

b) la reacción opuesta por la pared a los pies del hombre.

382. Un cuerpo de 200 N de peso se halla en equilibrio sobre un plano inclinado bajo la acción
de una fuerza F paralela al plano. Despreciando los rozamientos, calcule:

a) la intensidad de F ;

b) la intensidad de la fuerza normal que el cuerpo ejerce sobre el plano inclinado.

383. El bloque A de la figura pesa 100 N. El bloque B pesa 20 N y el sistema se halla en


equilibrio. ¿Cuál es la intensidad de la fuerza de rozamiento ejercida sobre el bloque A?

384. Un bloque A de 10 kg se encuentra en reposo sobre un plano horizontal liso, como se


m
indica en la figura de al lado. Considerando que las poleas y los hilos son ideales y g 10 ,y
s2
sabiendo que la masa del bloque C que equilibra el sistema es de 2 kg, calcule la masa del
bloque B.

Hidrostática
La hidrostática tiene su origen en los estudios de Arquímedes (287-212 a.C.) sobre la Mecánica
de los fluidos.

Si se comprendió la Unidad IV, se comprenderán fácilmente los principios de la Hidróstatica,


pues ahora, la fuerza será ejercida por un líquido o gas, o sea, un fluido.

Dieron contribución a este tema los científicos Torricelli (1608-1647), Stevin (1548-1620),
Pascal (1623-1662), entre otros.

La hidrostática ayuda a entender, por ejemplo, por qué los esquimales utilizan un zapato con
suela en forma de raqueta de tenis, qué es la presión atmosférica y cómo medirla, la diferencia
entre nadar en una piscina de agua dulce y en el mar, el funcionamiento de una prensa
hidráulica.

Al tomar una gaseosa con pajita, ¿por qué el líquido sube por la pajita? Al sumergir cuerpos en
líquidos, algunos se hunden y otros quedan flotando en la superficie. ¿Por qué será? ¿Y por
qué razón, cuando se sumerge a grandes profundidades se tienen problemas de respiración
aun con equipo de buceo?
122
Instituto Grigory Perelman. Prof. Master Emilio Ortiz
Trepowski. Teléfono 441.320. emiliortiz1@hotmail.com.

¿Cómo un avión a chorro, o aun movido a hélice, consigue volar? ¿Qué fuerzas lo mantienen
en el aire?

Fluido
Se denomina fluido a toda sustancia que puede fluir, es decir, deslizarse con facilidad. Por eso
a los líquidos y a los gases se denominan fluidos.

El estudio de los fluidos en reposo o en movimiento se trata en un apartado de la Física a que


se denomina Mecánica de los Fluidos.

Con fines didácticos la Mecánica de los Fluidos se divide en:

Fluido-estática: estudia los fluidos en reposo.

Fluido-dinámica: estudia los fluidos en movimiento.

Como el líquido utilizado antiguamente era el agua, cuyo prefijo es hidro, se emplean también
los nombres Hidrostática para la Fluido-estática e Hidrodinámica para la Fluido-Dinámica.

Densidad absoluta o masa específica


Se denomina densidad absoluta o masa específica de un cuerpo al cociente entre la masa y el
volumen del cuerpo.

m
V

Donde:

masa especifica
m masa del cuerpo
V volumen del cuerpo

La unidad de densidad absoluta en el Sistema Internacional es el kilogramo por metro cúbico,


kg
que se representa por .
m3

g
Es muy utilizada la unidad .
cm3

Observación:

123
Instituto Grigory Perelman. Prof. Master Emilio Ortiz
Trepowski. Teléfono 441.320. emiliortiz1@hotmail.com.

La densidad de un cuerpo puede no tener el mismo valor de la densidad absoluta de la


sustancia que constituye el cuerpo. Los valores serán iguales solamente cuando el cuerpo es
macizo y homogéneo.

Problema de aplicación
g m
La densidad de la glicerina es 1,26 3
. ¿Cuánto pesan 4l de glicerina? Considere g 10 2 .
cm s

Solución

g
1, 26
cm3

V 4l 4000cm3

m
g 10
s2

La masa de la glicerina es igual a:

m m
1, 26 m 5, 04kg
V 4000
m
Por lo tanto, P m g 5, 04kg 10 50, 4N
s2

Problemas Propuestos
411 Halle la densidad absoluta en g/cm3 de un cuerpo de forma cúbica que mide 10 cm de
arista y cuya masa es de 2 kg.

Solución.

m 2000 g
2
V 103 cm3

412 El cuerpo que se indica en la figura tiene una forma de paralelepípedo y su masa es de 6
kg. Determine su densidad absoluta en g/cm3 .

124
Instituto Grigory Perelman. Prof. Master Emilio Ortiz
Trepowski. Teléfono 441.320. emiliortiz1@hotmail.com.

Solución.

m 6000 g
500
V 12 cm3

413 La densidad de un cuerpo es de 1,8 g/cm 3 y su volumen es igual a 10 cm3 . Determine la


masa de ese cuerpo.

Solución.

m g
m V 1,8 3
10cm3 18g
V cm

g
414 Un cuerpo de 4 kg de masa tiene una densidad absoluta de 5 . Determine su
cm3
volumen en cm3 .

Solución.

m m 4000
V 800cm 3
V 5

415 La densidad de una determinada sustancia es 1,25 g/cm3 . Un cubo macizo, homogéneo,
tiene 2 cm de arista y se halla en un lugar donde la aceleración local de la gravedad es igual a
980 cm/s 2 . Calcule, en unidades del Sistema Internacional, la masa del cubo y su peso.

Solución.

125
Instituto Grigory Perelman. Prof. Master Emilio Ortiz
Trepowski. Teléfono 441.320. emiliortiz1@hotmail.com.

g g 1kg 1000000cm3 kg
1, 25 1, 25 1, 25 103
cm3 3
cm 1000g 1m 3 m3

a 2cm 0, 02m 2 10 2 m

cm cm 1m m
g 980 980 9,8
s2 s 2 100cm s2

m m m m
1, 25 103 1, 25 103 m 0, 01kg
V a3 2 3 8 10 6
2 10
P mg P 0, 01 9,8 0, 098N

Respuesta: 0,01 kg y 0,098 N.

416 En un proceso de electrodeposición de estaño, se produce una película de 1 10 4 m de


espesor. ¿Cuántos metros cuadrados se pueden cubrir con 1,46 kg de estaño, cuya masa
kg
específica es 7,3 103 ?
m3

Solución.

Diseñando la película se tiene:

126
Instituto Grigory Perelman. Prof. Master Emilio Ortiz
Trepowski. Teléfono 441.320. emiliortiz1@hotmail.com.

417 Un cilindro de hierro, vacío internamente, tiene 4 cm de radio externo y 2 cm de radio


g
interno. Sabiendo que la altura del cilindro es 10 cm y la densidad absoluta es 7,8 ,
cm3
calcule la masa del cilindro. Considere 3,1.

Solución.

127
Instituto Grigory Perelman. Prof. Master Emilio Ortiz
Trepowski. Teléfono 441.320. emiliortiz1@hotmail.com.

418. Se mezclan dos líquidos, A y B. El líquido A tiene un volumen de 120 cm3 y una densidad
g
absoluta de 0,78 3
. El líquido B tiene un volumen de 200 cm3 y una densidad igual a 0,56
cm
g g
3
. Determine en la densidad de la mezcla.
cm cm3

Solución.

VA 120cm3
g
A 0, 78
cm3
VB 200cm3
g
B 0,56
cm3

128
Instituto Grigory Perelman. Prof. Master Emilio Ortiz
Trepowski. Teléfono 441.320. emiliortiz1@hotmail.com.

mA
A
VA
mA 120 0, 78
mA 93, 6g
mB
B
VB
mB 0,56 200
mB 112g
mA mB 205, 6 g
0, 64
VA VB 320 cm3

g
Respuesta: 0, 64
cm3

Fórmula matemática de la presión


Considere dos personas, A y B, de igual peso, que caminan en la nieve.

Observe que la persona A deja marcas más profundas en la nieve que la persona B, a pesar de
que tienen el mismo peso.

Esto ocurre porque el área de la superficie sobre la cual A se apoya es menor que la superficie
de apoyo de B.

La física relaciona esas dos magnitudes, fuerza peso y área de la superficie, a través de la
magnitud física llamada presión.

Considerando la fuerza F que actúa perpendicularmente sobre el área S en la figura de al


lado.

129
Instituto Grigory Perelman. Prof. Master Emilio Ortiz
Trepowski. Teléfono 441.320. emiliortiz1@hotmail.com.

Se denomina presión (p) al cociente entre la intensidad de la fuerza F y el área S en el que la


fuerza se distribuye.

F
p
S
p presion
F fuerza
S area

La unidad de presión en el Sistema Internacional de Unidades es el newton por metro


cuadrado, que se indica por N/m 2 y es denominada pascal.

1N
1 pascal 1 Pa
m2

Otras unidades que también se utilizan son:

dyn
1) La , denominada baria:
cm2

1dyn
1baria 1ba
1cm 2

2) La atmósfera:

N dyn
1atm 760mmHg 105 106
m2 cm 2

Problema de Aplicación
Un cubo homogéneo de aluminio de 2 m de arista se halla apoyado en una superficie
kg m
horizontal. Sabiendo que la densidad del aluminio es 2, 7 103 3
y g 10 2 , ¿cuál
m s
es la presión ejercida por el bloque en la superficie?

Solución

a 2m
kg
2, 7 103
m3
m
g 10
s2

130
Instituto Grigory Perelman. Prof. Master Emilio Ortiz
Trepowski. Teléfono 441.320. emiliortiz1@hotmail.com.

La masa del cubo es igual a:

V a3 23 8m 3

m m
2, 7 103 m 2,16 104 kg
V 8

El área de apoyo es:

S a2 4m 2

F P mg 2,16 104 10 N
p 5, 4 10 4
S S S 4 m2

Problemas Propuestos
N
419. Determine, en 2
, la presión media ejercida por un edificio de 300t y 200 m 2 de base en
m
m
los puntos de contacto con el suelo. Adopte g 10 2 .
s

Solución.

m 300ton 300 103 kg


m
Datos p 3 106 N 300 103 kg 10 m a
s2
S 200m 2

Conceptos

peso 3 106 N
p 1,5 104 1,5 104 pascales
S 200 m2

420. Una bailarina de 50 kg de masa se apoya en la punta de un pie. La superficie de contacto


m N
entre el pie y el suelo tiene un área de 4 cm 2 . Siendo g 10 2
, determine en 2 la presión
s m
ejercida por el pie de la bailarina.

Solución.

131
Instituto Grigory Perelman. Prof. Master Emilio Ortiz
Trepowski. Teléfono 441.320. emiliortiz1@hotmail.com.

m 50kg
Datos P 500N
S 4cm 2 4 10 4 m 2 0, 0004m 2
P 500 N
p 1250000 1, 25 106
S 4 10 4 m2

421. Las dimensiones de un bloque de mármol son 0,5m 0,5m 2m . Una de sus caras
cuadradas se apoya sobre un plano horizontal. Calcule la presión ejercida sobre la cara
g m
apoyada, sabiendo que la masa específica del mármol es 2,8 3
y g 10 2 .
cm s

Solución.

g kg
2,8
3
2,8 103 3
cm m
Datos
m
g 10 2
s
El volumen del bloque esta dado por
V abc 0,5 0,5 2 0, 5m3

La masa del bloque es igual a

m m
2,8 103 m 1, 4 103 kg
V 0, 5

La presión es igual a

F mg 1, 4 103 10 N
p p 5, 6 106
S ab 0,5 0,5 m2

422. Una placa rectangular de vidrio, que mide 1 m de ancho y 2,5 m de longitud, se halla
N
sumergida en un líquido, en una región donde la presión es 10 en todos los puntos de la
m2
placa. ¿Cuál es, en newtons, la intensidad de la fuerza que actúa sobre esa cara de la placa?

Solución.

El área de la placa es igual a:

S 1 2,5 2,5m 2

132
Instituto Grigory Perelman. Prof. Master Emilio Ortiz
Trepowski. Teléfono 441.320. emiliortiz1@hotmail.com.

Por tanto,

F F
p 10 F 25N
S 2,5

m
423. Cuando el viento alcanza una velocidad de 45 en grandes tempestades, se ejerce una
s
N
presión de 28 aproximadamente. Determine la fuerza que él ejerce sobre una pared de
cm2
10 m de longitud y 2 m de altura, colocada perpendicularmente a la dirección del viento.

Solución.

N
p 28
Datos cm 2
S 2 10 20m 2 20 104 cm 2

F
p F 28 20 104 5, 6 106 N
S

424. Considere un bolígrafo de 0,08 N de peso y:

Sección de la base=0,5 cm 2

Sección de la punta=0,2 mm 2

Longitud=10 cm

Ancho de la base longitudinal de apoyo=0,2 mm

Determine la presión ejercida por el bolígrafo en cada uno de los tres casos que se indican en
la figura.

Solución.

133
Instituto Grigory Perelman. Prof. Master Emilio Ortiz
Trepowski. Teléfono 441.320. emiliortiz1@hotmail.com.

425. Cuatro cubos metálicos homogéneos e iguales, de 10 1 m de arista, se hallan dispuestos


sobre un plano horizontal, como se indica en la figura.

N m
Sabiendo que la presión aplicada por el conjunto al plano es 104 2
y g 10 2 , calcule la
m s
densidad de los cubos.

134
Instituto Grigory Perelman. Prof. Master Emilio Ortiz
Trepowski. Teléfono 441.320. emiliortiz1@hotmail.com.

426. Un cubo hueco de aluminio posee 100 g de masa y 50 cm3 de volumen. El volumen de la
parte vacía es de 10 cm3 . Calcule la densidad del cubo y la masa específica del aluminio.

427. Calcule la presión que la fuerza F de 12 N de intensidad, indicada en la figura, ejerce


sobre el área S 8cm 2 .

135
Instituto Grigory Perelman. Prof. Master Emilio Ortiz
Trepowski. Teléfono 441.320. emiliortiz1@hotmail.com.

Presión ejercida por un líquido

Considere el recipiente de la figura, que contiene un líquido de densidad absoluta


hasta una altura h, en un lugar donde la aceleración de la gravedad es g.

El líquido contenido en el recipiente tiene un peso P y ejerce sobre la base del


recipiente una presión p, dada por:

P mg Vg
p gh p gh
S V V
h h

Esa presión, debida solamente a la columna del líquido, también es denominada


presión hidrostática y puede ser aplicada a un punto cualquiera del recipiente.

Observación:

Si se tienen, por ejemplo, tres líquidos no miscibles dentro de un mismo


recipiente, la presión en el fondo es la suma de las presiones parciales que cada
líquido ejerce individualmente.

p fondo pA pB pC A ghA B ghB C ghC

Problema de Aplicación
El recipiente de la figura contiene agua hasta una altura de 20 cm.

Sabiendo que el área de la base vale, aproximadamente, 8 cm 2 y considerando


m g
g 10 y H 2O 1 , calcule:
s2 cm3

a) La presión ejercida por la columna de agua en el fondo del recipiente.

b) La fuerza que el agua ejerce en el fondo del recipiente.

Solución

Problemas Propuestos
136
Instituto Grigory Perelman. Prof. Master Emilio Ortiz
Trepowski. Teléfono 441.320. emiliortiz1@hotmail.com.

428. Calcule la presión y la fuerza en el fondo de los recipientes que se indican en


m
las figuras. Siendo g 10 .
s2

429. Un reservorio de agua posee en el fondo una válvula de 6 cm de diámetro. La


válvula se abre por acción del agua cuando ésta alcanza 1,8 m encima del nivel de
kg
la válvula. Suponiendo que la densidad del agua es 103 y la aceleración local
m3
m
de la gravedad es 10 , calcule la fuerza necesaria para abrir la válvula.
s2

430. Las dimensiones de una piscina de fondo plano horizontal de un club social
son: L 25m de longitud y l 10m de ancho. Sabiendo que el agua que la llena
ejerce una fuerza de 4,5 106 N en su fondo, determine la profundidad de esa
piscina. Siendo:

g
1 densidad del agua
cm3
m
g 10 2
s

431. Se dispone de dos recipientes cilíndricos: uno de diámetro D1 0, 6m y otro


de diámetro D2 0, 4m, ambos con altura suficiente para contener 150 litros de
aceite. Se sabe que el fondo de los recipientes es frágil y por eso se debe
almacenar el aceite en el cilindro que permitirá la menor presión hidrostática en el
fondo.

a) ¿Cuál de los recipientes se debe utilizar? ¿Por qué?

g
b) Sabiendo que aceite 0,8 , ¿cuál será la menor presión posible en el
cm3
fondo?

m
Considere g 10
s2

432. Halle la presión ejercida por los líquidos en el fondo del recipiente que se
indica en la figura.

Siendo:

137
Instituto Grigory Perelman. Prof. Master Emilio Ortiz
Trepowski. Teléfono 441.320. emiliortiz1@hotmail.com.

g
H 2O 1
cm3
g
Hg 13, 6
cm3
m
g 10
s2

433. Se colocan tres líquidos no miscibles en el interior de un vaso cilíndrico. El


g g
volumen y la densidad de cada líquido son: 0,6 l, 1,2 3
; 1,4 l, 0,8 ; y 0,9 l,
cm cm3
g
0,7 . ¿Cuál es la intensidad de la fuerza total que actúa sobre el fondo del
cm3
recipiente? Desprecie la presión atmosférica.

Presión atmosférica
Alrededor de la Tierra hay una capa de aire denominada atmósfera, cuya altura es
del orden de 18 km. Esa masa ejerce una presión sobre todos los cuerpos en su
interior, denominada presión atmosférica.

Cálculo de la presión atmosférica


Para determinar el valor de la presión atmosférica, Torricelli utilizó un tubo de 1 m
de longitud, lleno de mercurio.

Colocando la extremidad libre del tubo en un recipiente conteniendo mercurio, al


nivel del mar y a 0 C, el verificó que la columna de mercurio en el tubo alcanzó 76
cm.

Como los puntos A y B están en el mismo nivel y en el mismo líquido, se tiene:

pA pB Hg gh patm
13, 6 103 9,8 0, 76 patm
N
patm 1, 01 105
m2

Para efectuar los cálculos, se utiliza:

N
patm 1, 0 105
m2
138
Instituto Grigory Perelman. Prof. Master Emilio Ortiz
Trepowski. Teléfono 441.320. emiliortiz1@hotmail.com.

Como la columna de mercurio mide 76 cm, se puede escribir:

1atm 76cmHg 760mmHg presion atmosferica normal

La relación es:

1atm 106 dyn/cm 2 105 N/m 2 76cmHg

Conforme a la experiencia de Torricelli, cada cm 2 de la superficie terrestre soporta


una columna de atmósfera de aproximadamente 1 kg.

Las fuerzas ejercidas sobre los cuerpos por la atmósfera son considerables. Ellas
nos pasan desapercibidas porque nuestra presión interna es del mismo valor y
compensa sus efectos. Se puede comprobarlo a través de una experiencia muy
simple: calentando un poco de agua en el interior de una lata y tapándola cuando
el agua estuviera caliente, la presión atmosférica producirá la deformación de la
lata con el enfriamiento.

En un ambiente cerrado, por ejemplo, en una habitación, el aire ejerce la misma


presión en todos los puntos, independientemente de la altura en que estos se
encuentran. Esa presión gaseosa es debida al número de colisiones de las
partículas con las superficies (las paredes de la habitación). Tal presión es
constante.

Por otro lado, en la atmósfera el aire es un gas libre porque ella no posee paredes
laterales ni techo, solamente una pared de fondo –la superficie de la Tierra.

Como las moléculas gaseosas están sujetas a las fuerzas gravitacionales, ellas
tienden a estar más junto a la superficie terrestre. Por eso, la densidad del aire
disminuye con la altitud, es decir, el aire queda desaglomerado. Por lo tanto,
disminuye también la presión atmosférica, pero no uniformemente, conforme se
indica en la tabla.

Altitud (metros) Presión (mmHg)

0 760,0

200 741,1

2000 593,8

10000 193,0

16201 73,0

139
Instituto Grigory Perelman. Prof. Master Emilio Ortiz
Trepowski. Teléfono 441.320. emiliortiz1@hotmail.com.

20022 40,0

Teorema de Stevin
Considerando un líquido de densidad absoluta , en equilibrio en el recipiente de
la figura.

Sean los puntos A y B del líquido situados a una distancia hA y hB ,


respectivamente, de la superficie del líquido.

Las presiones debidas a la columna del líquido en esos puntos son:

pA hA g 1
pB hB g 2

Efectuando (1) – (2), se tiene:

pA pB hA g hB g pA pB g hA hB
pA pB g hA hB g h
pA pB g h

Observaciones:

1) Si los puntos A y B están en el mismo nivel (igual profundidad), se tiene que


h 0 , esto es, p A pB . En base a esta conclusión, se puede verificar que la
superficie libre de un líquido en equilibrio es siempre plana y horizontal, pues
todos sus puntos soportan la misma presión, debiendo entonces estar en el
mismo nivel.

2) Si el punto B está en la superficie del líquido, la presión ejercida por el aire es la


presión atmosférica. De ahí se concluye que el nivel de la superficie libre en los
vasos comunicantes es igual en todos los vasos, porque la presión en todos
ellos es la presión atmosférica.

3) En la igualdad p A pB g h, se tiene:
pA presion absoluta o presion total del punto A ; g h presión
debida a la columna h del líquido (presión hidrostática).

140
Instituto Grigory Perelman. Prof. Master Emilio Ortiz
Trepowski. Teléfono 441.320. emiliortiz1@hotmail.com.

Problemas de Aplicación

El recipiente de la figura contiene dos líquidos no miscibles, A y B.

g g m N
Sabiendo que A 1, 4 y B 0, 6 , g 10 2 y patm 105 ,
cm3 cm 3
s m2
calcule la presión total en el fondo del recipiente.

Solución

La presión en el fondo del recipiente es:

N
px patm A ghA B ghB 105 1, 4 103 10 0, 4 0, 6 10 3 10 0, 2 1, 067 10 5
m2

Los vasos comunicantes que se indican en la figura contienen los líquidos A y B


en equilibrio.

g g
Siendo A 0,8 y B 1, 4 , calcule el valor de x.
cm3 cm3

Teorema de Pascal
Cuando se ejerce una presión sobre un cuerpo sólido, ésta se transmite
desigualmente en distintas direcciones debido a la fuerte cohesión que da al
sólido su rigidez. En un líquido la presión se transmite igualmente en todas las
direcciones, debido a la fluidez.

Para demostrar el Teorema de Pascal, se consideran dos puntos, A y B, en el


interior de un líquido incompresible en equilibrio de densidad absoluta , en
un lugar donde la aceleración de la gravedad es igual a g.

La diferencia de presión entre los puntos A y B es:

pA pB gh

Aumentando la presión en los puntos A y B por cualquier proceso, los puntos A


y B experimentarán un aumento de presión p A y pB , tal que las presiones
son:

pA pA pA
pB pB pB

141
Instituto Grigory Perelman. Prof. Master Emilio Ortiz
Trepowski. Teléfono 441.320. emiliortiz1@hotmail.com.

Como el líquido es incompresible, la distancia entre los puntos A y B es la


misma. Luego:

pA pB gh
pA pA pB pB gh

De las ecuaciones anteriores tenemos que:

pA pB pA pA pB pB pA pB

Por lo tanto, se puede enunciar el Teorema de Pascal:

El aumento de presión ejercido en un punto de un líquido ideal en equilibrio se


transmite con igual intensidad a todos los puntos de ese líquido.

Las fotos ilustran algunas aplicaciones del Teorema de Pascal.

Prensa Hidráulica
Es una aplicación del Teorema de Pascal.

Consiste en dos cilindros verticales de secciones desiguales A y B, unidos por


un tubo en el interior del cual existe un líquido que contiene dos émbolos de
áreas S1 y S 2 .

Aplicando la fuerza F 1 al émbolo de área S1 se produce un aumento de


presión que se transmite a todos los puntos del líquido, inclusive a aquellos en
contacto con el émbolo del área S 2 . Luego:

F1 F2
p1 p2
S1 S2

Se observa que la prensa hidráulica efectúa una multiplicación de fuerza,


porque el área S 2 es mayor que el área S1 .

Problema de Aplicación

142
Instituto Grigory Perelman. Prof. Master Emilio Ortiz
Trepowski. Teléfono 441.320. emiliortiz1@hotmail.com.

Una prensa hidráulica tiene dos émbolos de áreas iguales a 10 cm 2 y 80 cm 2 .


Calcule la fuerza transmitida al émbolo mayor cuando se aplica al menor una
fuerza de 120 N.

Empuje
Cuando se intenta hundir una esfera de plástico en un líquido se observa que
cuanto más se hunde la esfera, mayor es la fuerza de resistencia ofrecida por
el líquido.

Esto ocurre porque el líquido ejerce sobre la esfera una fuerza vertical de
abajo hacia arriba llamada empuje.

El empuje representa la fuerza resultante del líquido en equilibrio sobre la


esfera.

Cálculo del empuje (teorema de Arquímedes)


Considerando un líquido en equilibrio y una porción de ese líquido como si
fuese un cuerpo sumergido en él.

Para que el cuerpo de masa mc y volumen Vc quede en equilibrio en el líquido,


se debe tener el empuje igual peso del cuerpo, es decir:

E P E mc g
E c Vc g

Pero el cuerpo imaginario está constituido del líquido. Luego:

c liquido y Vc Vliquido desplazado

Por lo tanto:

E liquido Vliquido desplazado g

Todo cuerpo sumergido total o parcialmente en un líquido recibe un empuje


vertical hacia arriba, igual al peso del líquido desplazado por el cuerpo.

Equilibrio de cuerpos sumergidos y flotantes


Considerando un cuerpo sumergido en un líquido. Se sabe que sólo dos
fuerzas actúan sobre él: su peso P y el empuje E .

143
Instituto Grigory Perelman. Prof. Master Emilio Ortiz
Trepowski. Teléfono 441.320. emiliortiz1@hotmail.com.

Se distinguen tres casos, que se verán a continuación:

1º caso: El peso es mayor que el empuje P E .

En este caso el cuerpo bajará con aceleración constante (condiciones ideales).


Si se verifican las expresiones de P y E, se concluye que eso sucederá si la masa
específica del cuerpo es mayor que la masa específica del líquido, es decir
c L .

2º caso: El peso es menor que el empuje P E .

En este caso el cuerpo subirá con aceleración constante hasta que quede
flotando en la superficie del líquido. Eso sucede cuando la masa específica del
cuerpo es menor que la masa específica del líquido, es decir, c L .

Cuando el cuerpo, en su trayectoria de subida, emerge en la superficie del


líquido, el empuje comienza a disminuir, pues disminuye la parte sumergida y,
por lo tanto, el volumen del líquido se desplaza. El cuerpo subirá hasta que el
empuje quede igual al peso del cuerpo, que es constante. En esa condición
(P=E), el cuerpo quedará en equilibrio, flotando en el líquido.

3º caso: El peso es igual al empuje P E .

En este caso el cuerpo queda en equilibrio, cualquiera que sea el punto en que
se lo coloque. Eso sucede cuando la masa específica del cuerpo es igual a la
masa específica del líquido, es decir, c L .

Problemas de Aplicación

1. Un cubo de madera de densidad absoluta 0,2 g/ cm3 y arista de 20 cm flota


en el agua. Determine la altura de la parte sumergida del cubo.

Solución

Se debe tener el empuje igual al peso. Luego:

144
Instituto Grigory Perelman. Prof. Master Emilio Ortiz
Trepowski. Teléfono 441.320. emiliortiz1@hotmail.com.

E P liquido Vliquido desplazado g mc g


liquido Vliquido desplazado mc
liquido S h1 c S h
liquido h1 c h
1 h1 0, 2 20
h1 4cm

2. Una esfera de 20 g de masa se mantiene totalmente sumergida en un


líquido, de forma que la distancia entre su punto más alto y la superficie
libre del líquido es 11,25 cm. Sabiendo que la densidad de la esfera en
relación al líquido es 0,8, determine el tiempo transcurrido desde el
instante en que la esfera es liberada hasta que ella llega a la superficie. Se
m
admite la inexistencia de rozamiento y g 10 .
s2

Solución

Cálculo del empuje:

m 1
E liq V g E liq g E 0, 020 10 E 0, 25 N
esf 0,8

Cálculo del peso de la esfera:

P m g P 0, 020 10 0, 2N

Cálculo de la aceleración (MUV):

m
E P mesf a 0, 25 0, 2 0, 020 a a 2,5
s2

Cálculo del tiempo de subida:

1 2 1
s s0 v0 t at s 2,5t 2 1, 25t 2
2 2

Siendo s 11, 25cm 0,1125m, se tiene:

0,1125 1, 25t 2 t2 0, 09 t 0,3s

145
Instituto Grigory Perelman. Prof. Master Emilio Ortiz
Trepowski. Teléfono 441.320. emiliortiz1@hotmail.com.

Problemas Propuestos
449. Se coloca, dentro de un tanque con agua de 1g/cm3 , un cuerpo de
500 g de masa y 1 000 ml de volumen, que queda flotando en la superficie
del agua con la mitad de su volumen sumergido. ¿Cuál es la intensidad en
N (newtons) del empuje que ejerce el agua sobre el cuerpo?

450. Un iceberg de forma cúbica flota emergiendo 1 m de su altura.


Determine la altura de la parte sumergida, sabiendo que la densidad
absoluta del hielo es de 0,9 g/ cm3 y la densidad del agua salada es de 1,01
g/ cm3 .

1
451. Una esfera maciza y homogénea flota en el agua emergiendo de su
5
volumen. Calcule la densidad del material de que está hecha la esfera.
Adopte agua 1g/cm3 .

452. Un cubo macizo de madera con lado de longitud L=0,5 m flota en el


agua, conservando dos caras horizontales. Calcule la altura de la parte
sumergida del cubo, siendo m masa específica de la
madera 7 102 kg/m3 y masa específica del agua= 1 103 kg/m 3 .

453. Un cuerpo sólido flota en el agua (masa específica 1,00 g/ cm3 ) con
60% de su volumen sumergido. Calcule el porcentaje del volumen del
cuerpo sólido que está emergiendo cuando se coloca un líquido de masa
específica 0,8 g/ cm3 .

454. Dos cuerpos de densidades diferentes pero de la misma forma son


colocados dentro de un recipiente que contiene líquido y, después de
algún tiempo, quedan en la posición descrita en la figura. Sabiendo que el
cuerpo indicado por la letra A tiene 320 g de masa y 400 cm3 de volumen,
determine la densidad del cuerpo B.

455. Una esfera de densidad absoluta x flota entre los líquidos A y B de


densidades absolutas d A 0, 7g/cm3 y d B 0, 9g/cm 3 , conforme lo
indica la figura. La superficie de separación de los líquidos pasa por el
centro de la esfera. Calcule el valor de x.

456. Dentro de un vaso abierto se colocan 2 kg de agua. Después se coloca


dentro del líquido un pequeño cuerpo de 500 g de masa y 50cm 2 de

146
Instituto Grigory Perelman. Prof. Master Emilio Ortiz
Trepowski. Teléfono 441.320. emiliortiz1@hotmail.com.

volumen, colgado por un hilo, conforme se indica en la figura. Calcule la


intensidad de tracción en el hilo.

457. Un estudiante quiere conocer el volumen de un cuerpo irregular de


una mezcla desconocida cuya masa es igual a 0,45 kg. Para alcanzar su
objetivo, él realiza el montaje de la figura.

Siendo: masa específica del agua=103 kg/m3 , lectura en el


dinamómetro= 2,1N , g 10m/s 2 . Calcule el volumen del cuerpo.

Solución

458. Un balón para estudio atmosférico tiene 50 kg de masa (incluyendo el


gas), 110 m3 de volumen y está sujeto a la tierra por medio de una cuerda.
En la ausencia del viento, la cuerda permanece estirada y vertical.

147
Instituto Grigory Perelman. Prof. Master Emilio Ortiz
Trepowski. Teléfono 441.320. emiliortiz1@hotmail.com.

Considerando la densidad del aire igual a 1,3 kg/m3 y g=10 m/s 2 , calcule
la intensidad de tracción sobre la cuerda.

459. Una esfera cuyo volumen es de 200 cm3 , hecha de un material cuya
densidad es 0,8 g/cm 3 , está totalmente sumergida en un tanque lleno de
agua (densidad 1g/cm3 ) de profundidad 10 m y se la abandona en el
fondo. Considerando g 10m/s 2 y p0 105 N / m2 , calcule:

a) la presión que la esfera soporta en el fondo del tanque;

b) el módulo, la dirección y el sentido de la aceleración adquirida por la


esfera;

c) la velocidad de la esfera cuando alcanza la superficie del agua;

d) el tiempo que la esfera empleará para alcanzar la superficie del agua.

460. Un cuerpo de m=2 kg de masa está en reposo colgado de un hilo y se


encuentra totalmente sumergido en un líquido. La fuerza de tracción en el
hilo tiene una intensidad T=16 N. Calcule la aceleración adquirida por el
cuerpo inmediatamente después de que el operador corta el hilo. Adopte
g=10 m/s 2 .

461. Un bloque cúbico (arista=10 cm), colgado por una cuerda sujeta en el
extremo de una barra rígida, está sumergido en el agua (masa específica=1
g/cm3 ). Con la barra apoyada en un soporte, el sistema se mantiene en
equilibrio por la acción de una fuerza F (módulo=25 N) aplicada en el otro
extremo, conforme lo indica la figura.

Considerando la aceleración de la gravedad igual a 10m/s 2 , realice lo


siguiente:

a) Represente las fuerzas que actúan sobre el bloque;

b) Halle el empuje sobre el bloque;


148
Instituto Grigory Perelman. Prof. Master Emilio Ortiz
Trepowski. Teléfono 441.320. emiliortiz1@hotmail.com.

c) Calcule el peso del bloque;

d) Determine la masa específica del bloque.

462. En una experiencia de laboratorio, los alumnos observaron que


una esfera de masa especial se hundía en el agua. Arquímedes, un
alumno creativo, puso sal en el agua y vio que la esfera flotó. Ulises
consiguió el mismo efecto modelando la masa bajo la forma de un
barquito. Explique, con argumentos de Física, los efectos observados
por Arquímedes y por Ulises.

Ejercicio de examen anterior

Un bloque de uranio de 10 N está suspendido de un dinamómetro y


sumergido en mercurio de 13, 6 103 kg/m 3 de densidad. La lectura del
dinamómetro es 2,9 N. Entonces, la masa específica del uranio es
aproximadamente:

a) 5,5 103 kg/m3

b) 24 103 kg/m3

c) 19 103 kg/m 3

d) 14 103 kg/m 3

e) 2 103 kg/m3

Solución

149
Instituto Grigory Perelman. Prof. Master Emilio Ortiz
Trepowski. Teléfono 441.320. emiliortiz1@hotmail.com.

150

Вам также может понравиться